ĐẠI HỌC QUỐC GIA HÀ NỘI
TRƯỜNG ĐẠI HỌC GIÁO DỤC
PHẠM THỊ LAN ANH
XÂY DỰNG HỆ THỐNG BÀI TẬP VÀ HƯỚNG DẪN HOẠT ĐỘNG
GIẢI BÀI TẬP CHƯƠNG CẢM ỨNG ĐIỆN TỪ-VẬT LÝ 11
NHẰM BỒI DƯỠNG HỌC SINH GIỎI VẬT LÝ Ở TRUNG HỌC PHỔ
THÔNG CHUYÊN
LUẬN VĂN THẠC SĨ SƯ PHẠM VẬT LÝ
HÀ NỘI – 2013
ĐẠI HỌC QUỐC GIA HÀ NỘI
TRƯỜNG ĐẠI HỌC GIÁO DỤC
PHẠM THỊ LAN ANH
XÂY DỰNG HỆ THỐNG BÀI TẬP VÀ HƯỚNG DẪN HOẠT ĐỘNG
GIẢI BÀI TẬP CHƯƠNG CẢM ỨNG ĐIỆN TỪ- VẬT LÝ 11
NHẰM BỒI DƯỠNG HỌC SINH GIỎI VẬT LÝ Ở TRUNG HỌC PHỔ
THÔNG CHUYÊN
LUẬN VĂN THẠC SĨ SƯ PHẠM VẬT LÝ
CHUYÊN NGÀNH LÝ LUẬN VÀ PHƯƠNG PHÁP DẠY HỌC
(BỘ MÔN VẬT LÝ)
MÃ SỐ: 601410
Cán bộ hướng dẫn: GS.TS. Nguyễn Quang Báu
HÀ NỘI – 2013
LỜI CẢM ƠN
Luận văn được hoàn thành dưới sự hướng dẫn tận tình của GS.TS. Nguyễn
Quang Báu. Em xin bày tỏ lòng biết ơn sâu sắc tới thầy. Xin trân trọng cảm ơn
Thầy đã nhiệt tình hướng dẫn em trong quá trình nghiên cứa và thực hiện đề tài.
Em xin chân thành cảm ơn Phòng đào tạo trường Đại học Giáo dục - Đại học
Quốc Gia Hà Nội đã tạo điều kiện thuận lợi cho em trong quá trình học tập và làm
luận văn tốt nghiệp.
Em xin chân thành cảm ơn các giảng viên của trường Đại học Giáo dục và
thầy cô trong khoa Vật lý Đại học KHTN Hà Nội cùng các thầy cô trong khoa Vật
lý trường Đại hạc Sư Phạm Hà Nội đã tận tình giảng dạy, giúp đỡ và đóng góp
nhiều ý kiến quý báu về mặt chuyên môn và phương pháp cho em trong quá trình
nghiên cứu và hoàn thành luận văn.
Xin chân thành cảm ơn Ban giám hiệu cùng các bạn đồng nghiệp trường
THPT Thanh Oai A đã giúp đỡ và trao đổi chuyên môn trong quá trình tôi học tập
và nghiên cứu.
Xin chân thành cảm ơn các thầy cô giáo trong tổ bộ môn Vật lý trường
THPT Chuyên – KHTN – Đại học KHTN – Đại học Quốc gia Hà Nội cùng các em
học sinh yêu quý đã tạo điều kiện giúp đỡ và cộng tác với tôi trong quá trình làm
thực nghiệp sư phạm và hoàn thành luận văn.
Hà Nội, ngày 05 tháng 12 năm 2013
Học viên
i
Phạm Thị Lan Anh
DANH MỤC CÁC CHỮ VIẾT TẮT
THPT
Trung học phổ thông
KHTN
Khoa học Tự nhiên
ĐHQG
Đại học Quốc gia
Nxb
Nhà xuất bản
HS
Học sinh
HSG
Học sinh giỏi
THCS
Trung học cơ sở
TNSP
Thực nghiệm sư phạm.
ii
MỤC LỤC
Lời cảm ơn ................................................................................................................. i
Danh mục các chữ viết tắt .......................................................................................... ii
Mục lục ...................................................................................................................... iii
Danh mục các bảng ................................................................................................... .vi
Danh mục các sơ đồ hình vẽ .................................................................................... vii
MỞ ĐẦU .................................................................................................................... 1
Chương 1:CƠ SỞ LÝ LUẬN VÀ THỰC TIỄN VỀ PHƯƠNG PHÁP HƯỚNG
DẪN GIẢI BÀI TẬP VẬT LÝ TRUNG HỌC PHỔ THÔNG VÀ BỒI DƯỠNG
HỌC SINH GIỎI VẬT LÝ....................................................................................... 5
1.1.Bồi dưỡng học sinh giỏi Vật lý và học sinh THPT chuyên .................................. 5
1.1.1. Học sinh giỏi Vật lý và học sinh THPT chuyên ............................................... 5
1.1.1.1. Học sinh giỏi và học sinh giỏi Vật lý ............................................................. 5
1.1.1.2. Học sinh THPT chuyên .................................................................................. 5
1.1.2. Giáo dục học sinh giỏi....................................................................................... 6
1.1.2.1. Một số quan điểm về giáo dục học sinh giỏi.................................................. 6
1.1.2.2. Mục tiêu dạy học sinh giỏi ............................................................................. 7
1.1.3. Những năng lực, phẩm chất cần có của học sinh giỏi ....................................... 7
1.1.3.1. Năng lực và phẩm chất cần có của học sinh giỏi nói chung .......................... 7
1.1.3.2. Năng lực và phẩm chất cần có của học sinh giỏi Vật lý ................................ 8
1.1.4. Một số biện pháp bồi dưỡng học sinh giỏi Vật lý ............................................ 9
1.1.4.1. Tổ chức giảng dạy ......................................................................................... 9
1.1.4.2. Hướng dẫn tự học ........................................................................................ 10
1.1.4.3. Rèn luyện năng lực tư duy và khả năng suy luận logic cho học sinh .......... 11
1.1.4.4.Thi giải toán học sinh giỏi từng học kỳ ........................................................ 11
1.2. Bài tập Vật lý trong dạy học ở trường trung học phổ thông .............................. 12
1.2.1. Khái niệm vai trò và mục đích sử dụng của bài tập Vật lý ............................. 12
1.2.1.1. Khái niệm bài tập Vật lý .............................................................................. 12
1.2.1.2. Vai trò, mục đích sử dụng bài tập Vật lý trong dạy học .............................. 12
1.2.2. Phân loại bài tập Vật lý ................................................................................... 13
1.2.2.1. Phân loại theo nội dung ................................................................................ 13
1.2.2.3. Phân loại theo yêu cầu mức độ phát triển tư duy ......................................... 15
iii
1.2.4. Phương pháp giải bài tập Vật lý ...................................................................... 15
1.2.4.1. Lựa chọn bài tập Vật lý ................................................................................ 15
1.2.4.2. Phương pháp giải bài tập Vật lý ................................................................... 16
1.2.5. Các kiểu hướng dẫn giải bài tập Vật lý ........................................................... 17
1.2.5.1. Hướng dẫn theo mẫu .................................................................................... 17
1.2.5.2. Hướng dẫn tìm tòi ........................................................................................ 18
1.2.5.3. Định hướng khái quát chương trình hóa ...................................................... 18
1.3. Tình hình thực tế công tác bồi dưỡng học sinh giỏi Vật lý ở Trường Trung học
phổ thông Chuyên Tự nhiên ...................................................................................... 18
1.3.1. Đội ngũ giáo viên Vật lý và thành tích của học sinh giỏi Vật lý ở trường
THPT chuyên KHTN ................................................................................................ 18
1.3.2.Thực trạng công tác bồi dưỡng học sinh giỏi Vật Lý ở trường THPT chuyên
KHTN ........................................................................................................................ 19
1.3.2.1. Thuận lợi trong công tác bồi dưỡng học sinh giỏi ở trường THPT chuyên
KHTN ........................................................................................................................ 19
1.3.2.2.Một số yêu tố bất thuận lợi trong trong công tác bồi dưỡng học sinh giỏi ở
trường THPT chuyên KHTN .................................................................................... 20
KẾT LUẬN CHƯƠNG I .......................................................................................... 22
Chương 2: XÂY DỰNG HỆ THỐNG BÀI TẬP VÀ HƯỚNG DẪN HOẠT
ĐỘNG GIẢI BÀI TẬP CHƯƠNG CẢM ỨNG ĐIỆN TỪ – VẬT LÝ 11 ......... 24
2.1. Nội dung kiến thức chương Cảm ứng điện từ .................................................... 24
2.1.1. Cấu trúc nội dung chương Cảm ứng điện từ ................................................... 24
2.1.2. Phân tích sơ lược nội dung chương Cảm ứng điện từ ..................................... 25
2.1.2.1. Hiện tượng cảm ứng điện từ......................................................................... 25
2.1.2.2. Hiện tượng tự cảm ........................................................................................ 29
2.2. Mục tiêu chương Cảm ứng điện từ: ................................................................... 31
2.2.1. Kiến thức: ........................................................................................................ 31
2.2.2. Kỹ năng. .......................................................................................................... 31
2.3. Phương pháp xây dựng và hướng dẫn giải hệ thống bài tập chương Cảm ứng
điện từ. ....................................................................................................................... 31
2.3.1. Phương pháp xây dựng hệ thống bài tập chương Cảm ứng điện từ. ............... 31
2.3.2. Phương pháp hướng dẫn giải bài tập chương Cảm ứng điện từ ..................... 32
2.3.2.1. Phương pháp hướng dẫn giải bài tập định tính chương Cảm ứng điện từ ... 32
iv
2.3.2.2. Phương pháp hướng dẫn giải bài tập định lượng chương Cảm ứng điện từ ........ 32
2.3.2.3. Mục đích của việc hướng dẫn giải bài tập chương Cảm ứng điện từ .......... 34
2.4. Xây dựng hệ thống bài tập và hướng dẫn hoạt động giải bài tập chương ......... 35
2.4.1. Bài tập định tính .............................................................................................. 35
2.4.1.1. Bài tập có hướng dẫn ................................................................................... 35
2.4.1.2. Bài tập tự giải ............................................................................................... 38
2.4.2. Bài tập định lượng ........................................................................................... 40
2.4.2.1. Bài tập có hướng dẫn ................................................................................... 40
2.5.1.1. Bài tập tự giải ............................................................................................... 50
KẾT LUẬN CHƯƠNG II ......................................................................................... 56
Chương 3: THỰC NGHIỆM SƯ PHẠM .............................................................. 57
3.1. Mục đích, đối tượng và phương pháp của thực nghiệm sư phạm ...................... 57
3.1.1. Mục đích của thực nghiệm sư phạm. .............................................................. 57
3.1.2. Đối tượng và phương pháp của thực nghiệm sư phạm ................................... 57
3.1.2.1. Đối tượng: .................................................................................................... 57
3.1.2.2. Phương thức thực nghiệm sư phạm ............................................................. 57
3.1.2.3 Tiêu chí đánh giá kết quả thực nghiệm sư phạm .......................................... 58
3.1.2.4. Thời gian tiến hành thực nghiệm ................................................................. 59
3.2. Tiến hành thực nghiệm sư phạm ........................................................................ 59
3.3. Kết quả và xử lý kết quả .................................................................................... 60
3.3.1. Phân tích định tính diễn biến các giờ học trong quá trình TNSP .................... 60
3.3.2. Phân tích bài kiểm tra ...................................................................................... 61
3.4. Đánh giá chung về thực nghiệm sư phạm .......................................................... 65
KẾT LUẬN CHƯƠNG 3 ........................................................................................ 66
KẾT LUẬN VÀ KHUYẾN NGHỊ ......................................................................... 67
1. Kết luận ................................................................................................................. 67
2. Khuyến nghị .......................................................................................................... 68
TÀI LIỆU THAM KHẢO ...................................................................................... 69
v
PHỤ LỤC ................................................................................................................ 70
DANH MỤC CÁC BẢNG
Bảng 3.1: Bảng xếp loại học tập theo các mức ............................................. 59
Bảng 3.2. Bảng thống kê điểm số .......................................................... 62
Bảng 3.3. Bảng các tham số thống kê ..................................................... 62
vi
Bảng 3.4. Bảng thống kê số học sinh đạt từ điểm xi trở xuống ............ 62
DANH MỤC CÁC SƠ ĐỒ HÌNH VẼ
Sơ đồ 1.1.Cấu trúc nội dung chương Cảm ứng điện từ .................................... 25
Hình 3.1. Đồ thị đường phân bố tần suất ......................................................... 63
vii
Hình 3.2. Đồ thị đường phân bố tần suất lũy tích hội tụ lùi ............................ 63
MỞ ĐẦU
1. Lý do chọn đề tài
Giáo dục được coi là quốc sách hàng đầu của nước ta, hiện nay Ngành đang
có những đổi mới và được quan tâm đặc biệt. Trong đó, đào tạo nhân tài vẫn là một
trong những mục tiêu hàng đầu của ngành giáo dục, các trường chuyên là một trong
những mũi nhọn tiên phong trong quá trình đào tạo nhân tài cho đất nước. Hệ thống
các trường chuyên trung học phổ thông trên cả nước đã và đang đóng vai trò quan
trọng trong việc phát hiện, bồi dưỡng học sinh giỏi và là cái nôi để đào tạo các nhà
khoa học, nhà quản lý và các doanh nhân giỏi.
Đào tạo học sinh Chuyên, học sinh giỏi ở bậc Trung học Phổ thông (THPT)
là một quá trình mang tính khoa học đòi hỏi phải có chiến lược lâu dài và có
phương pháp phù hợp.
Trong quá trình giảng dạy ở trường phổ thông nhiệm vụ phát triển tư duy cho
học sinh là nhiệm vụ rất quan trọng, nhiệm vụ này cần được tiến hành đồng bộ ở
các môn, trong đó Vật lý là môn khoa học tự nhiên đề cập đến nhiều vấn đề của
khoa học, đời sống sẽ góp phần rèn luyện tư duy cho học sinh ở mọi góc độ, đặc
biệt là qua phần giải bài tập Vật lý.
Sử dụng hệ thống bài tập Vật lý là một trong các biện pháp quan trọng để
nâng cao chất lượng HSG Vật lý. Đối với học sinh giải bài tập là một cách học tập
tích cực, là phương tiện chuyển tải kiến thức, rèn luyện tư duy và kỹ năng thực hành
bộ môn hiệu quả nhất. Bài tập Vật lý không những có tác dụng rèn luyện kỹ năng
vận dụng, đào sâu và mở rộng kiến thức đã học một cách sinh động, phong phú mà
còn thông qua đó để ôn tập, rèn luyện một số kỹ năng cần thiết cho việc học tập và
nghiên cứu Vật lý rèn luyện tính tích cực, tự lực, trí thông minh sáng tạo cho học
sinh, giúp học sinh hứng thú trong học tập. Cũng thông qua bài tập Vật lý giáo viên
kiểm tra, đánh giá việc nắm vững kiến thức và kỹ năng Vật lý của học sinh.
Trong các lớp chuyên Vật lý trung học phổ thông của nước ta hiện nay, học
sinh được luyện nhiều bài tập khó dẫn đến quen, còn nặng về tính toán đôi khi chưa
phát huy được óc quan sát, khả năng phát hiện vấn đề và các năng lực tư duy cho
học sinh. Còn thiếu những nghiên cứu và hướng dẫn chi tiết cho công tác bồi dưỡng
80
học sinh giỏi Vật lý ứng với từng chương bài và chủ đề cụ thể.
Vì các lí do trên nên tôi quyết định chọn đề tài nghiên cứu "Xây dựng hệ
thống bài tập và hướng dẫn hoạt động giải bài tập chương Cảm ứng điện từ –
Vật lý 11 nhằm bồi dưỡng học sinh giỏi Vật lý ở Trung học phổ thông chuyên"
với mục đích góp phần nâng cao chất lượng bồi dưỡng học sinh giỏi, nâng cao chất
lượng giảng dạy Vật lý ở các lớp chuyên Vật lý THPT hiện nay.
2. Mục đích nghiên cứu
Xây dựng hệ thống bài tập và hướng dẫn hoạt động giải bài tập chương Cảm
ứng điện từ – Vật lý 11 nhằm bồi dưỡng học sinh giỏi Vật lý ở THPT chuyên.
3. Nhiệm vụ nghiên cứu
- Tìm hiểu đặc điểm, năng lực của Học sinh giỏi, Học sinh Chuyên Vật lý
THPT
- Tìm hiểu lý luận về phương pháp bồi dưỡng học sinh giỏi, học sinh giỏi Vật
lí ở trường THPT
- Tìm hiểu lý luận về vai trò, tác dụng, phương pháp giải và hướng dẫn giải bài
tập Vật lý.
- Nghiên cứu nội dung kiến thức chương Cảm ứng điện từ - Vật lý 11 THPT.
- Lựa chọn, xây dựng hệ thống bài tập chương Cảm ứng điện từ - Vật lý 11
THPT.
- Định hướng, xây dựng phương pháp giải bài tập và hướng dẫn hoạt động giải
bài tập chương Cảm ứng điện từ - Vật lý 11 Trung học phổ thông.
- Thực nghiệm sư phạm để đánh giá ưu điểm, nhược điểm và hiệu quả của hệ
thống bài tập và phương pháp hướng dẫn hoạt động giải bài tập chương cảm
ứng điện từ.
4. Khách thể và đối tượng nghiên cứu
- Khách thể nghiên cứu: Học sinh Chuyên Lý lớp 11 THPT chuyên.
- Đối tượng nghiên cứu: Hệ thống bài tập và phương pháp hướng dẫn hoạt
động giải bài tập chương Cảm ứng điện từ – Vật lý 11 – Trung học phổ thông.
5. Vấn đề nghiên cứu
Xây dựng hệ thống bài tập và hướng dẫn hoạt động giải bài tập chương Cảm
ứng điện từ – Vật lý 11 - Trung học phổ thông như thế nào sẽ bồi dưỡng được học
80
sinh giỏi Vật lý ở trung học phổ thông chuyên?
6. Giả thuyết khoa học
Xây dựng được hệ thống bài tập chương “Cảm ứng điện từ- Vật lý 11” gồm
nhiều dạng bài, có mức độ khó, tổng hợp nhiều mảng kiến thức. Kết hợp với
việc hướng dẫn hoạt động giải bài tập theo các phương pháp có sự định hướng
tư duy cho học sinh sẽ góp phần bồi dưỡng học sinh giỏi Vật lý ở Trung học phổ
thông chuyên.
7. Giới hạn và phạm vi nghiên cứu
- Áp dụng với chương Cảm ứng điện từ – Vật lý 11 - Trung học phổ thông.
- Nghiên cứu cho học sinh học môn Vật lý ở khối chuyên Vật l1- Trung học
phổ thông Chuyên Khoa học Tự nhiên.
8. Ý nghĩa khoa học và thực tiễn
- Ý nghĩa lý luận: Tìm hiểu những đặc điểm và yêu cầu cần có của học sinh
giỏi, học sinh chuyên Lý Trung học phổ thông và đặc điểm, vai trò của bài tập vật
lý. Từ đó biên soạn hệ thống bài tập chương Cảm ứng điện từ và áp dụng các
phương pháp hướng dẫn giải bài tập phù hợp giúp bồi dưỡng học sinh giỏi Vật lý.
- Ý nghĩa thực tiễn: Kết quả nghiên cứu có thể được áp dụng trong việc
giảng dạy chương Cảm ứng điện từ - Vật lý 11 Trung học phổ thông ở các trường
THPT chuyên khác trong cả nước. Đồng thời nó còn có giá trị tham khảo cho các
thầy cô ở các trường THPT khi luyện tập cho học sinh giỏi để tham gia các kỳ thi
học sinh giỏi các cấp.
9. Phương pháp nghiên cứu
- Phương pháp nghiên cứu lý thuyết: Tham khảo sách báo, tạp chí chuyên
ngành, sưu tầm tài liệu về bài tập Vật lý, phương pháp hướng dẫn giải bài tập
Vật lý và vai trò của bài tập Vật lý trong dạy học.
- Phương pháp nghiên cứu thực tiễn: Phương pháp thực nghiệm, phương pháp
điều tra
- Phương pháp thống kê toán học.
10. Cấu trúc luận văn
Ngoài phần Mở đầu, kết luận, tài liệu tham khảo, phụ lục, nội dung chính của
luận văn được trình bày trong 03 chương.
80
Chương 1: Cơ sở lý luận và thực tiễn về dạy bài tập Vật lý phổ thông.
Chương 2: Hệ thống bài tập và phương pháp hướng dẫn hoạt động giải bài
tập chương Cảm ứng điện từ - Vật lý 11
80
Chương 3: Thực nghiệm sư phạm.
CHƯƠNG 1
CƠ SỞ LÝ LUẬN VÀ THỰC TIỄN VỀ PHƯƠNG PHÁP HƯỚNG DẪN GIẢI
BÀI TẬP VẬT LÝ TRUNG HỌC PHỔ THÔNG VÀ BỒI DƯỠNG HỌC
SINH GIỎI VẬT LÝ
1.1.Bồi dưỡng học sinh giỏi Vật lý và học sinh THPT chuyên
1.1.1. Học sinh giỏi Vật lý và học sinh THPT chuyên
1.1.1.1. Học sinh giỏi và học sinh giỏi Vật lý
“Học sinh giỏi là học sinh chứng minh được trí tuệ ở trình độ cao và có khả
năng sáng tạo, thể hiện một động cơ học tập mãnh liệt và đạt xuất sắc trong lĩnh vực
lý thuyết, khoa học, cần một sự giáo dục đặc biệt và sự phục vụ đặc biệt để đạt được
trình độ tương ứng với năng lực [16, tr.1]
Cũng có nhiều nước quan niệm: Học sinh giỏi là những đứa trẻ có năng lực
trong các lĩnh vực trí tuệ, sáng tạo, nghệ thuật và năng lực lãnh đạo hoặc lĩnh vực lý
thuyết.
Học sinh giỏi Vật lý là người có năng lực quan sát tốt, nắm vững bản chất
của hiện tượng Vật lý mong muốn khám phá các hiện tượng Vật lý và vận dụng tối
ưu các kiến thức thức Vật lý để giải quyết một hay nhiều vấn đề mới, bài tập mới có
thể chưa được học hoặc thấy bao giờ.
Nói chung học sinh giỏi và học sinh giỏi Vật lý đều có đặc điểm là tích cực,
chủ động, sáng tạo trong học tập có năng lực tư duy phát triển. Trong qua trình dạy
học để lựa chọn phương pháp dạy học phù hợp giáo viên luôn phải quan tâm đến
những đặc điểm này.
1.1.1.2. Học sinh THPT chuyên
Học sinh được tuyển chọn vào các trường THPT chuyên hầu hết là các
em có tư chất thông minh, ham học hỏi, tích cực và chủ động trong quá trình
học tập, nghiên cứu. Chương trình và phương pháp dạy học cho học sinh ở
trường THPT chuyên được xây dựng và lựa chọn phải căn cứ vào đặc điểm đó
và mục tiêu của trường THPT chuyên: “Mục tiêu của trường chuyên là phát hiện
những học sinh có tư chất thông minh, đạt kết quả xuất sắc trong học tập và phát
triển năng khiếu của các em về một số môn học trên cơ sở đảm bảo giáo dục phổ
80
thông toàn diện; giáo dục các em thành người có lòng yêu nước, tinh thần vượt khó,
tự hào, tự tôn dân tộc; có khả năng tự học, nghiên cứu khoa học và sáng tạo; có sức
khỏe tốt để tiếp tục đào tạo thành nhân tài đáp ứng yêu cầu phát triển đất nước” [3,
tr.2]
1.1.2. Giáo dục học sinh giỏi
1.1.2.1. Một số quan điểm về giáo dục học sinh giỏi
Trong chiến lược phát triển giáo dục phổ thông. Nhiều nước ghi riêng thành
một mục dành cho HSG hoặc coi đó là một dạng của giáo dục đặc biệt. Trong thực
tế đã có những phân tích về quá trình phát triển của giáo dục học sinh giỏi trên thế
giới:
“Trên thế giới việc phát hiện và bồi dưỡng HSG đã có từ rất lâu. Ở Trung
Quốc, từ đời nhà Đường những trẻ em có tài đặc biệt được mời đến sân Rồng để
học tập và được giáo dục bằng những hình thức đặc biệt.
Trong tác phẩm phương Tây, Plato cũng đã nêu lên các hình thức giáo dục
đặc biệt cho HSG. Ở châu Âu trong suốt thời Phục hưng, những người có tài năng
về nghệ thuật, kiến trúc, văn học... đều được nhà nước và các tổ chức cá nhân bảo
trợ, giúp đỡ.
Nước Mỹ mãi đến thế kỉ 19 mới chú ý tới vấn đề giáo dục học sinh giỏi và
tài năng. Đầu tiên là hình thức giáo dục linh hoạt tại trường St. Public Schools
Louis 1868 cho phép những HSG học chương trình 6 năm trong vòng 4 năm; sau đó
lần lượt là các trường Woburn; Elizabeth; Cambridge…Trong suốt thế kỉ XX, HSG
đã trở thành một vấn đề của nước Mỹ với hàng loạt các tổ chức và các trung tâm
nghiên cứu, bồi dưỡng học sinh giỏi ra đời. Năm 2002 có 38 bang của Hoa Kỳ có
đạo luật về giáo dục HSG (Gifted & Talented Student Education Act) trong đó 28
bang có thể đáp ứng đầy đủ cho việc giáo dục học sinh giỏi.
Từ năm 2001 chính quyền New Zealand đã phê chuẩn kế hoạch phát triển
chiến lược HSG. Cộng hòa liên bang Đức có Hiệp hội dành cho HSG và tài năng
Đức...
Giáo dục Phổ thông Hàn Quốc có một chương trình đặc biệt dành cho HSG
nhằm giúp chính quyền phát hiện học sinh tài năng từ rất sớm. Năm 1994 có
khoảng 57/ 174 cơ sở giáo dục ở Hàn Quốc tổ chức chương trình đặc biệt dành cho
80
HSG.
Từ năm 1985, Trung Quốc thừa nhận phải có một chương trình GD đặc biệt
dành cho hai loại đối tượng HS yếu kém và HSG, trong đó cho phép các HSG có
thể học vượt lớp.” [16, tr. 2]
Như vậy hầu như tất cả các nước đều coi trọng vấn đề đào tạo và bồi dưỡng
học sinh giỏi trong chiến lược phát triển chương trình GD phổ thông. Nhiều nước
ghi riêng thành một mục dành cho HSG, một số nước coi đó là một dạng của giáo
dục đặc biệt hoặc chương trình đặc biệt.
1.1.2.2. Mục tiêu dạy học sinh giỏi
Mục tiêu chính của chương trình dành cho HSG nhìn chung ở các nước đều
tương đối giống nhau. Có thể nêu lên một số điểm chính sau đây:
- Phát triển phương pháp suy nghĩ ở trình độ cao phù hợp với khả năng trí tuệ của
trẻ.
- Bồi dưỡng khả năng lao động, làm việc sáng tạo
- Phát triển các kĩ năng, phương pháp và thái độ tự học suốt đời
- Nâng cao ý thức và khát vọng của trẻ về sự tự chịu trách nhiệm
- Khuyến khích sự phát triển về lương tâm và ý thức trách nhiệm trong đóng góp xã
hội
- Phát triển phẩm chất lãnh đạo.
1.1.3. Những năng lực, phẩm chất cần có của học sinh giỏi [5, tr. 1 - 15]
1.1.3.1. Năng lực và phẩm chất cần có của học sinh giỏi nói chung
Học sinh giỏi cần có một số năng lực và phẩm chất quan trọng sau:
- Năng lực tiếp thu kiến thức
+ Khả năng nhận thức vấn đề nhanh, rõ ràng
+ Luôn hứng thú trong các tiết học, đặc biệt là bài mới
+ Có ý thức tự bổ sung, hoàn thiện những tri thức đã thu được ngay từ dạng sơ khai
- Năng lực suy luận
+ Biết phân tích sự vật, hiện tượng qua các dấu hiệu đặc trưng của chúng
+ Biết thay đổi góc nhìn khi xem xét một sự vật hiện tượng
+ Biết cách tìm con đường ngắn nhất để đi đến một kết luận chính xác
+ Biết xét đủ các điều kiện cần thiết để kết luận được giả thuyết đúng
80
+ Biết quay lại điểm vừa xuất phát để tìm đường đi mới
- Năng lực đặc biệt
+Biết diễn đạt chính xác điều mình muốn trình bày
+ Sử dụng thành thạo hệ thống ký hiệu, các quy ước để diễn tả vấn đề
+ Biết phân biệt thành thạo các kỹ năng đọc, viết và nói
+ Biết thu gọn các vấn đề và trật tự hóa các vấn đề để dùng khái niệm trước, mô tả
cho khái niệm sau
- Năng lực lao động sáng tạo:
Biết tổ hợp các yếu tố, các thao tác để thiết kế một dãy các hoạt động nhằm
đạt kết quả mong muốn
- Năng lực kiểm chứng:
+ Biết suy xét đúng sai từ một loạt sự kiện
+ Biết tạo ra các tương tự hay tương phản để khẳng định hoặc bác bỏ một đặc trưng
nào đó trong sản phẩm do mình làm ra
+ Biết chỉ ra một cách chắc chắn các dữ liệu cần phải kiểm nghiệm khi thực hiện
một số lần kiểm nghiệm
- Năng lực thực hành:
+ Biết thực hiện dứt khoát một số động tác trong khi làm thí nghiệm
+ Biết kiên trì, kiên nhẫn trong quá trình làm sáng tỏ một số vấn đề lý thuyết thông
qua thực nghiệm hoặc đi đến một vấn đề lý thuyết mới.
1.1.3.2. Năng lực và phẩm chất cần có của học sinh giỏi Vật lý
Dựa trên nguyên tắc chung về phẩm chất của một học sinh giỏi và đặc điểm
của môn Vật lý có thể khái quát các năng lực và phẩm chất của một học sinh giỏi
Vật lý THPT như sau.
- Có năng lực tư duy tốt và sáng tạo, biết phân tích, tổng hợp, so sánh, khái quát
hóa.
- Có kiến thức Vật lý vững vàng, sâu sắc, hệ thống, chính là nắm vững bản chất của
các hiện tượng Vật lý. Biết vận dụng linh hoạt, sáng tạo kiến thức cơ bản trong các
tình huống mới.
- Có kỹ năng thí nghiệm tốt, có năng lực về phương pháp nghiên cứu khoa học Vật
80
lý. Biết nêu ra những lý luận cho những hiện tượng xảy ra trong thực tế, biết cách
dùng thí nghiệm để kiểm chứng lại lý luận và biết cách dùng lý thuyết để giải thích
những hiện tượng đã được kiểm chứng.
Như vậy đối với giáo viên, khi đào tạo những học sinh giỏi Vật lý, cần
hướng học sinh học tập để học sinh được trang bị những kiến thức, kỹ năng, giúp
các em tự học hỏi, sáng tạo nhằm phát huy tối đa năng lực của mình.
1.1.4. Một số biện pháp bồi dưỡng học sinh giỏi Vật lý
1.1.4.1. Tổ chức giảng dạy [6,tr. 168]
Đào tạo các học sinh giỏi là cả một quá trình đào tạo nghiêm túc khoa học và
công phu. Trong quá trình này đòi hỏi chúng ta phải xây dựng được lộ trình đào tạo
hàng năm cho nhóm, lớp học sinh giỏi. Các nhóm học sinh này không chỉ được học
theo chương trình phổ thông hiện hành mà còn phải bổ sung thêm kiến thức theo
chương trình thi học sinh giỏi theo chương trình của khu vực và quốc tế, vì vậy cần
nghiên cứu để học sinh có thể đáp ứng theo chuẩn đào tạo trong nước mà còn
hướng tới chuẩn khu vực và quốc tế. Từ việc xây dựng chương trình chi tiết riêng
cho các nhóm học sinh giỏi. Các giai đoạn thực hiện có thể như sau:
Giai đoạn 1 : Củng cố kiến thức ở THCS. Các em học sinh được tổng kết các
kiến thức ở THCS, học một số kiến thức ở lớp 10, làm quen với một số chuyên đề
bồi dưỡng học sinh giỏi.
Giai đoạn 2 : Chuyên đề nâng cao. Các em trong nhóm học sinh giỏi ngoài
học các kiến thức cơ bản trên lớp còn học thêm các kiến thức cần thiết cho học sinh
giỏi (những kiến thức không có trong chương trình phổ thông). Ở trên lớp các em
hoàn thành chương trình của lớp 10 – 11 cơ bản đủ để học các chuyên đề bồi dưỡng
và giải các bài tập khó. Các học sinh lớp 10 trong nhóm luyện học sinh giỏi hoàn
thành các nội dung kiến thức nâng cao cho đội tuyển.
Giai đoạn 3 : Thành đội tuyển thi học sinh giỏi quốc gia và quốc tế.
Các học sinh đội tuyển 11 sẽ tham dự 3 kỳ thi học sinh giỏi cùng với học sinh lớp
12 để chọn đội tuyển. Các trường chọn các học sinh có điểm cao nhất lập thành đội
tuyển thi quốc gia và quốc tế. Đối với các học sinh giỏi các mục tiêu nhận thức từ
bậc 2 (áp dụng, phân tích, tổng hợp) đến bậc 3 (đánh giá) được rèn luyện ở thứ bậc
80
cao nhất.
1.1.4.2. Hướng dẫn tự học
“Đối với các tài liệu khoa học lại cần phải biết cách đọc. Đối với học sinh
giỏi càng cần nghiên cứu sâu vào chuyên ngành. Cần tập luyện cho học sinh tự học
mở rộng được kiến thức và bồi dưỡng được trí tuệ.
Rèn luyện cho học sinh cách đọc có hệ thống:
Học có hệ thống còn là học theo lịch sử phát triển của vấn đề. Bất cứ vấn đề
nào cũng có nguồn gốc phát sinh, quá trình phát triển trong đó nó liên quan đến
nhiều vấn đề khác và bản thân nó cũng qua một quá trình đấu tranh phức tạp của
mâu thuân nội tại, để đi đến hình thái ta thấy ngày nay.
Học có hệ thống còn là xem xét mỗi sự vật, mỗi hiện tượng trong mối tương
quan với xung quanh nó, xem xét các khía cạnh của sự vật cũng như các mặt của
hiện tượng.
Học có hệ thống còn là khi học một ngành khoa học phải biết cả cac nghành
có liên quan. Không thể học nguyên Vật lí mà thiếu khái niệm cơ bản về toán học.
Không thể giỏi về Vật lí, nếu không có kiến thức về toán học, hóa học, sinh học.
Học có hệ thống là học phải đi đôi với hành, lý thuyết phải gắn liền với thực
tập. Trong mọi quá trình đào tạo, phải đảm bảo cho sự cân đối nêu trên.
Rèn luyện cho học sinh cách lập kế hoạch và học có kế hoạch:
Học có kế hoạch trước hết là chia giáo trình thành phần. quy định thời gian
dành cho mỗi phần và tìm mọi cách để thanh toán từng phần theo thời gian đã định.
Sau phần đầu phải rút kinh nghiệm để, nếu cần, điều chỉnh thời gian dành cho các
phần sau thích hợp hơn.
Học có kế hoạch còn là nhằm từng vấn đề trong thời kỳ nhất định rồi đặt kế
hoạch đọc sách vở, tài liệu liên quan tới vấn đề đó. Khi chọn vấn đề cũng phải theo
đúng quy trình logic của nhận thức, nhằm những vấn đề dễ trước và vấn đề khó sau.
Khi đọc tài liệu cũng phải đọc từ nông tới sâu, từ khái quát tới chi tiết.
Biết cách học còn là biết lưu ý tới thứ tự ưu tiên của kiến thức: Điểm trung
tâm của vòng kiến thức phải là phương pháp luận (phương pháp tư duy, phương
pháp làm việc, phương pháp nghiên cứu…) lúc này hơn bao giờ hết, phương pháp
80
có vai trò quyết định trong kết quả học tập của từng người.” [6, tr. 170]
1.1.4.3. Rèn luyện năng lực tư duy và khả năng suy luận logic cho học sinh
Tư duy là một quá trình nhận thức khái quát và gián tiếp những sự vật và
hiện tượng của hiện thực trong những dấu hiệu, những thuộc tính bản chất của
chúng, những mối quan hệ khách quan, phổ biến giữa chúng, đồng thời cũng là sự
vận dụng sáng tạo những kết luận khái quát đã thu được vào những dấu hiệu cụ
thể, dự đoán được những thuộc tính, hiện tượng, quan hệ mới.“Năng lực tư duy là
tổng hợp những khả năng ghi nhớ, tái hiện, phân tích, tổng hợp, trừu tượng hóa,
khái quát hóa, tưởng tượng, suy luận, sáng tạo- giải quyết vấn đề, xử lý và linh
cảm trong quá phản ánh, phát triển tri thức và vận dụng chúng vào thực tiễn.”[12,
tr. 70]. Như Anhxtanh khẳng định “cái đích cuối cùng của dạy học là phát triển tư
duy cho học sinh” và bồi dưỡng học sinh giỏi cũng vậy, đặc biệt cần thiết phải rèn
luyện nâng cao năng lực tư duy bậc cao cho học sinh.
Căn cứ vào mục tiêu giáo dục cho học sinh giỏi đòi hỏi giáo viên tổ chức các
hoạt động dạy và học để phát triển các năng lực tư duy bậc cao ở học sinh như tư
duy lí luận, logic và sáng tạo. Trong quá trình bồi dưỡng học sinh giỏi, ở từng giai
đoạn giáo viên có nhiệm vụ làm xuất hiện những tình huống bắt buộc học sinh
phải thực hiện các thao tác tư duy và hành động nhận thức mới có thể giải quyết
được vấn đề và hoàn thành được nhiệm vụ học tập:
+ Giáo viên đưa ra những bài tập, câu hỏi để định hướng cho học sinh tìm những
thao tác tư duy hay phương pháp suy luận, hành động trí tuệ thích hợp.
+ Giáo viên phân tích câu trả lời hoặc bài tập của học sinh, chỉ ra chỗ sai của họ
trong khi thực hiện các thao tác tư duy và hướng dẫn cách sửa chữa.
+ Giáo viên giúp học sinh khái quát hoá kinh nghiệm thực hiện các suy luận
lôgic dưới dạng những qui tắc đơn giản.
1.1.4.4.Thi giải toán học sinh giỏi từng học kỳ
“Mỗi học kỳ khối chuyên tổ chức một cuộc thi giải các bài tập khó được sưu
tầm từ các tài liệu hay. Những em có lời giải đúng nhiều nhất, hay nhất và kết quả
tốt nhất sẽ được khen thưởng. Tổng kết các kết quả, lời giải của học sinh nhà trường
có một tài liệu bổ ích lưu lại cho các em khóa sau. Để đặt được kết quả cao trong
các kỳ thi đó các em phải nỗ lực rèn luyện các kỹ năng làm bài, phải tự học, qua đó
80
sẽ đạt được các mục tiêu học tập ở mức độ cao” [6, tr. 171]
1.2. Bài tập Vật lý trong dạy học ở trường trung học phổ thông
1.2.1. Khái niệm vai trò và mục đích sử dụng của bài tập Vật lý
1.2.1.1. Khái niệm bài tập Vật lý
Bài tập vật lí được hiểu là một vấn đề được đặt ra đòi hỏi phải giải quyết nhờ
những suy luận lôgic, những phép toán và thí nghiệm dựa trên cơ sở các khái niệm,
các thuyết, các định luật và các phương pháp vật lí. Bài tập Vật lý, là một phần hữu
cơ của quá trình dạy học Vật lý vì nó cho phép hình thành và làm phong phú các
khái niệm Vật lý, phát triển tư duy Vật lý và thói quen vận dụng kiến thức Vật lý
vào thực tiễn
1.2.1.2. Vai trò, mục đích sử dụng bài tập Vật lý trong dạy học
“Trong quá trình dạy học ở trung học phổ thông bài tập Vật lý đóng vai trò
đặc biệt quan trọng. Ở mỗi góc độ, mỗi trường hợp cụ thể bài tập Vật lý đều thể
hiện có những vai trò đặc thù của mình:
- Trong qúa trình giải quyết các tình huống cụ thể do các bài tập vật lý đặt ra, học
sinh phải sử dụng các thao tác tư duy như phân tích, tổng hợp, so sánh, khái quát
hóa, trừu tượng hóa …để giải quyết vấn đề, do đó tư duy của học sinh có điều kiện
để phát triển. Việc rèn luyện, hướng dẫn học sinh giải bài tập Vật lý là biện pháp rất
hiệu quả phát triển tư duy Vật lý cho học sinh.
- Về phương diện giáo dục, giải các bài tập Vật lý sẽ giúp hình thành các phẩm chất
cá nhân của học sinh như tình yêu lao động, trí tò mò, sự khéo léo, khả năng tự lực,
hứng thú với học tập, ý chí và sự kiên trì đạt tới mục đích đặt ra.
- Giải các bài toán Vật lý góp phần giáo dục kỹ thuật tổng hợp cho học sinh. Các
bài tập Vật lý gắn với cuộc sống, với kỹ thuật, với thực tiễn sản xuất thường đem lại
nhiều hứng thú cho học sinh.
- Giải các bài tập Vật lý là một phương pháp đơn giản để kiểm tra, hệ thống hóa
kiến thức, kỹ năng và thói quen thực hành, cho phép mở rộng, làm sâu sắc kiến thức
đã học.
- Vận dụng các định luật Vật lý, đặc biệt là các định luật bảo toàn, sẽ tạo cơ hội
hình thành thế giới quan duy vật biện chứng cho học sinh, phát triển tư duy biện
80
chứng đồng thời với tư duy lôgic hình thức.” [14, tr. 7 - 11]
Để phát huy tối ưu các vai trò đó, trong quá trình dạy học giáo viên cần phải
chú ý lựa chọn, xây dựng hệ thống bài tập và phương pháp hướng dẫn giải bài tập
Vật lý đa dạng và phù hợp với mỗi mục đích sử dụng. Bài tập Vật lý có thể được sử
dụng với những mục đích cụ thể như sau:
- Ôn tập, củng cố, mở rộng kiến thức, kỹ năng cho học sinh
- Huy động kiến thức cũ, kiến thức nền tảng, đặt vấn đề khởi đầu kiến thức mới
- Phát triển tư duy lí luận, tư duy sáng tạo của học sinh và bồi dưỡng phương pháp
nghiên cứu khoa học cho học sinh.
- Rèn luyện kỹ năng, kỹ xảo vận dụng kiến thức lý thuyết vào thực tiễn, phát triển
thói quen vận dụng kiến thức một cách khái quát.
- Rèn luyện đức tính tự lập, kiên trì vượt khó và năng lực tự học của học sinh
- Kiểm tra mức độ nắm vững kiến thức và kỹ năng của học sinh.
1.2.2. Phân loại bài tập Vật lý
“Bài tập Vật lý có thể khác nhau về nội dung, về phương thức giải, về mục
tiêu dạy học. Do đó việc phân loại bài tập Vật lý được dựa trên các cơ sở phân loại
như sau: Phân loại theo nội dung, phân loại theo phương thức cho điều kiện và
phương thức giải, phân loại theo yêu cầu luyện tập kỹ năng, phát triển tư duy sáng
tạo của học sinh” [15, tr.12]
1.2.2.1. Phân loại theo nội dung
Bài tập có nội dung lịch sử : là các bài tập chứa đựng các kiến thức có liên
quan đến lịch sử như những dữ liệu về các thí nghiệm vật lí cổ điển, những phát
minh, hay những câu chuyện có tính chất lịch sử.
Bài tập có nội dung cụ thể hoặc trừu tượng. Trong đó bài tập có nội dung cụ
thể: là bài tập có dữ liệu là các con số cụ thể, thực tế và học sinh có thể đưa ra lời
giải dựa vào vốn kiến thức vật lí cơ bản đã có. Bài tập có nội dung trừu tượng: là
những bài tập mà các dữ kiện cho dưới dạng chữ. Trong bài tập này, bản chất được
nêu bật trong đề bài, những chi tiết không bản chất đã được lược bỏ bớt. Học sinh
có thể nhận ra được cần sử dụng công thức, định luật vật lí nào để giải bài tập đã
cho.
Đề tài vật lý: cơ học, nhiệt học, điện học, quang học.
80
Kỹ thuật tổng hợp: là các bài tập có nội dung chứa đựng các kiến
1.2.2.2. Phân loại theo phương thức cho điều kiện và phương thức giải [15, tr.
17-20]
Theo cách này, người ta phân các bài tập Vật lý thành 5 loại: Bài tập định
tính, bài tập định lượng, bài tập thí nghiệm, bài tập đồ thị. Phân loại này cho phép
giáo viên lựa chọn bài tập tương ứng với sự chuẩn bị toán học của học sinh, mức độ
kiến thức và sáng tạo của học sinh.
Bài tập định tính: Điểm nổi bật của bài tập định tính là ở chỗ trong điều kiện
của bài toán đều nhấn mạnh bản chất Vật lý của hiện tượng. Giải các bài tập định
tính bằng các lập luận logic trên cơ sở các định luật Vật lý.
Khi giải bài tập định tính, học sinh được rèn luyện tư duy logic, khả năng phân tích
hiện tượng, trí tưởng tượng khoa học, khả năng vận dụng kiến thức. Vì vậy luyện
tập tốt nhất nên bắt đầu từ bài tập định tính.
Bài tập định lượng: Bài tập định lượng là những bài tập khi giải phải sử dụng
các phương pháp toán học dựa trên các định luật, quy tắc hoặc thuyết Vật lý. Đây là
dạng bài tập phổ biến nhất, sử dụng rộng rãi trong chương trình Vật lý trung học
phổ thông. Dạng bài tập này có điểm nổi bật lớn nhất là khắc sâu kiến thức của học
sinh, rèn luyện cho học sinh những phương pháp nhận thức đặc thù của Vật lý, đặc
biệt là phương pháp suy luận toán học.
Bài tập thí nghiệm: Đây là dạng bài tập trong đó thí nghiệm là công cụ để tìm
ra các đại lượng trong bài toán, hoặc để kiểm chứng một định luật, một thuyết trong
Vật lý. Bài tập thí nghiệm có thể là thí nghiệm biểu diễn hoặc thí nghiệm thực tập
của học sinh.
Bài tập đồ thị: là dạng bài tập phân tích đồ thị từ đó tìm ra các điều kiện giải
bài toán. Dạng bài tập này chủ yếu rèn kỹ năng vẽ và đọc đồ thị cho học sinh. Việc
áp dụng phương pháp đồ thị cho phép diễn đạt trực quan hiện tượng Vật lý, cho
cách giải trực quan hơn, phát triển kỹ năng vẽ và đọc đồ thị là các kỹ năng rất cần
thiết trong kỹ thuật.
Bài tập trắc nghiệm khách quan: Bài tập dạng trắc nghiệm khách quan thường
dùng để kiểm tra kiến thức trong phạm vi rộng, số lượng người được kiểm tra
80
nhiều, kết quả thu được khách quan không phụ thuộc vào người chấm. Bài tập này
yêu cầu học sinh phải nhớ, hiểu và vận dụng đồng thời rất nhiều các kiến thức liên
quan.
1.2.2.3. Phân loại theo yêu cầu mức độ phát triển tư duy
Căn cứ vào mức độ hoạt động tư duy của học sinh trong quá trình tìm kiếm lời
giải bài tập Vật lý phân chia vài tập Vật lý thành hai loại: Bài tập cơ bản và bài tập
phức hợp
Bài tập cơ bản: Là loại bài tập Vật lý mà để tìm được lời giải chỉ cần xác lập
mối quan hệ trực tiếp, tường minh giữa những cái đã cho và một cái phải tìm chỉ
dựa vào một kiến thức cơ bản vừa học mà học sinh chỉ cần tái hiện kiến thức chứ
không thể tạo ra.
Bài tập tổng hợp: Là loại bài tập Vật lý trong đó việc tìm kiếm lời giải phải
thực hiện một chuỗi các lập luận logic, biến đổi toán học qua nhiều mối liên hệ giữa
những cái đã cho, cái phải tìm với những cái trung gian không cho trong dữ kiện bài
toán. Bản thân việc xác lập mối liên hệ trung gian đó là một bài tập cơ bản. Và do
đó, muốn giải được bài tập phức hợp buộc người học phải giải được thành thạo các
bài tập cơ bản, ngoài ra còn phải biết cách phân tích bài tập phức hợp để quy nó về
các bài tập đơn giản đã biết.
Cũng có các nhà sư phạm phân loại bài tập theo yêu cầu này và chia thành hai
mức bài tập luyện tập và bài tập sáng tạo, trong đó bài tập
1.2.4. Phương pháp giải bài tập Vật lý
1.2.4.1. Lựa chọn bài tập Vật lý
Do bài tập Vật lý có nhiều cách phân loại, nhiều mục đích sử dụng khác nhau
nên để phát huy hiệu quả các vai trò và tác dụng của bài tập Vật lý trong quá trình
dạy học việc lựa chọn bài tập Vật lý cần dựa trên nhưng cơ sở sau: Mục đích sử
dụng, trình độ xuất phát của học sinh và thời gian cho phép sử dụng
Việc lực chọn bài tập cần đáp ứng các yêu cầu về số lượng và nội dung như
sau:
+ Phù hợp với mức độ nội dung các kiến thức cơ bản và các kỹ năng giải bài tập.
+ Hệ thống bài tập bao gồm nhiều thể loại.
+ Chú ý thích đáng về số lượng và nội dung các bài tập nhằm giúp học sinh vượt
80
qua những khó khăn chủ yếu, khắc phục những sai lầm phổ biến.
+ Các bài tập đưa ra phải có tính hệ thống mỗi bài tập là một mắt xích trong toàn bộ
hệ thống bài tập
+ Các bài tập phải đảm bảo tính vừa sức đối với đa số học sinh , đồng thời có chú ý
tới sự phân hóa học sinh.
1.2.4.2. Phương pháp giải bài tập Vật lý
“Mặc dù các bài tập Vật lý khác nhau về loại và mục đích sử dụng trong dạy
học, song trong thực tế người ta cũng thừa nhận một quan điểm chung về một quá
trình giải một bài tập Vật lý. Theo quan điểm đó, người thầy không chỉ đơn giản
trình bày cho học sinh cách giải mà phải thực hiện nhiệm vụ giáo dục, giáo dưỡng
học sinh trong quá trình giải bài tập, cần dạy học sinh tự lực giải bài tập Vật lý” [15,
tr. 21 - 27] . Quá trình giải các bài tập Vật lý có thể chia thành các bước, quy trình
tiến hành cho từng dạng bài cụ thể. Tuy nhiên có thể khái quát thành bốn bước sau.
Bước 1- Tìm hiểu đề bài: Đọc đúng đề bài, mô tả hiện tượng vật lí nêu trong
đề bài (có thể vẽ hình), xác định xem trong lớp hiện tượng vật lí đã cho có những
đại lượng vật lí nào đã cho, đại lượng nào cần tìm.
Bước 2- Xây dựng lập luận:
Xây dựng lập luận trong giải bài tập định tính: Có thể làm theo hai cách với
hai loại bài tập định tính là giải thích hiện tượng và dự đoán hiện tượng
“Xây dựng lập luận trong giải bài tập giải thích hiện tượng
+ Giải thích hiện tượng thực chất là cho biết một hiện tượng và lí giải xem vì sao
hiện tượng lại xảy ra như thế.
+ Trong các bài tập này, bắt buộc phải thiết lập được mối quan hệ giữa hiện tượng
cụ thể với một số đặc tính của sự vật hay với một số định luật vật lí.
+ Thực hiện phép suy luận lôgic, luận ba đoạn trong đó tiền đề thứ nhất là một đặc
tính chung của sự vật hoặc định luật vật lí tổng quát, tiền đề thứ hai là những điều
kiện cụ thể, kết luận về hiện tượng được nêu ra.”[6, tr.104]
- Xây dựng lập luận trong giải bài tập dự đoán hiện tượng.
+ Dự đoán hiện tượng thực chất là căn cứ vào những điều kiện cụ thể của đề bài,
xác định những định luật chi phối hiện tượng và dự đoán được hiện tượng gì xảy ra
80
và xảy ra thế nào.
+ Ta thực hiện suy luận lôgic, thiết lập luận ba đoạn, trong đó ta mới biết tiền đề thứ
hai (phán đoán khẳng định riêng), cần phải tìm tiền đề thứ nhất (phán đoán khẳng
định chung) và kết luận (phán đoán khẳng định riêng).
+ Trong trường hợp hiện tượng xảy ra phức tạp, ta phải xây dựng một chuỗi luận ba
đoạn liên tiếp ứng với các giai đoạn diễn biến của hiện tượng.
Xây dựng lập luận trong bài toán định lượng: Có thể có hai phương pháp
xây dựng lập luận
- Phương pháp phân tích: “ Tìm một định luật, một quy tắc diễn đạt bằng một công
thức có chứa đại lượng cần tìm và một vài đại lượng khác chưa biết. Tiếp tục tìm
những định luật, công thức khác cho biết mối quan hệ giữa đại lượng chưa biết này
với các đại lượng đã biết trong đề bài. Cuối cùng tìm được một công thức chỉ chứa
đại lượng cần tìm với đại lượng đã biết.” [6, tr.105]
- Phương pháp tổng hợp: “Từ những đại lượng đã cho ở đề bài. Dựa vào các định
luật, quy tắc vật lí, tìm những công thức có chứa đại lượng đã cho với các đại lượng
trung gian mà ta dự kiến có liên quan đến đại lượng cần tìm. Suy luận toán học, đưa
đến công thức chỉ chứa đại lượng phải tìm với các đại lượng đã cho.” [6, tr.105]
Bước 3. Luận giải(tìm ra quy luật chung): Tính toán để tìm ra kết quả.
Bước 4. Biện luận: Phân tích kết quả cuối cùng để loại bỏ những kết quả không phù
hợp với điều kiện đầu bài tập hoặc không phù hợp với thực tế và kết luận
1.2.5. Các kiểu hướng dẫn giải bài tập Vật lý
Để hướng dẫn giải bài tâp cho học sinh hiệu quả, thì trước hết giáo viên phải
giải được bài tập đó, và phải xuất phát từ mục đích sư phạm để xác định kiểu hướng
dẫn cho phù hợp. Giáo viên cần xác định trước mục đích sử dụng bài tập, xác định
những kiến thức áp dụng để giải bài tập, phát hiện những khó khăn mà học sinh có
thể gặp khi giải bài tập và soạn sẵn câu hỏi hướng dẫn học sinh vượt qua khó khăn.
Giáo viên có thể lựa chọn hoặc kết hợp những kiểu hướng dẫn sau:
1.2.5.1. Hướng dẫn theo mẫu
Hướng dẫn angorit là sự hướng dẫn chỉ rõ cho học sinh những hành động cụ
thể cần thực hiện và trình tự thực hiện các hành động đó để đạt được kết quả mong
80
muốn.
Hướng dẫn angorit thường dùng khi cần dạy cho học sinh phương pháp giải một bài
toán điển hình, luyện cho học sinh kỹ năng giải một dạng bài tập xác định.
1.2.5.2. Hướng dẫn tìm tòi
Định hướng tìm tòi là kiểu định hướng mang tính chất gợi ý cho học sinh suy
nghĩ tìm tòi phát hiện cách giải bài toán. Hướng dẫn tìm tòi thường dùng khi cần
giúp đỡ học sinh vượt qua khó khăn để giải quyết được bài tập đồng thời vẫn đảm
bảo được yêu cầu phát triển tư duy học sinh muốn tạo điều kiện để học sinh tự lực
tìm tòi cách giải quyết
1.2.5.3. Định hướng khái quát chương trình hóa
Là định hướng tư duy của học sinh theo đường lối khái quát của việc giải
quyết vấn đề.
Kiểu hướng dẫn này áp dụng khi có điều kiện hướng dẫn tiến trình hoạt động giải
bài tập, nhằm giúp cho học sinh tự giải được bài tập đã cho, đồng thời dạy cho học
sinh cách suy nghĩ trong quá trình giải bài tập.
Trong quá trình hướng dẫn học sinh giải bài tập không thể theo một khuôn
mẫu nhất định, mà tùy thuộc vào nội dung, kiến thức, yêu cầu của bài toán, và còn
tùy thuộc vào đối tượng học sinh mà chúng ta có cách lựa chọn kiểu hướng dẫn cho
phù hợp. Đồng thời người giáo viên phải biết phối hợp cả ba kiểu hướng dẫn trên
để phát huy tối ưu tác dụng của việc hướng dẫn trong việc giải các bài tập tổng hợp.
1.3. Tình hình thực tế công tác bồi dưỡng học sinh giỏi Vật lý ở Trường Trung
học phổ thông Chuyên Tự nhiên
1.3.1. Đội ngũ giáo viên Vật lý và thành tích của học sinh giỏi Vật lý ở trường
THPT chuyên KHTN
“Trường THPT chuyên Khoa học Tự nhiên Hà Nội (HUS High School for Gifted
Students) là hệ đào tạo trung học phổ thông của trường Đại học Khoa học Tự nhiên
Hà Nội (Đại học Quốc gia Hà Nội). Các khối chuyên của Trường THPT chuyên
Khoa học Tự nhiên Hà Nội đóng góp nhiều thành viên cho đội tuyển học sinh giỏi
quốc gia và học sinh giỏi quốc tế. Trường cũng đồng thời được công nhận là ngôi
trường giàu thành tích nhất Việt Nam về số lượng huy chương cũng như giải thưởng
đạt được trong các kỳ thi học sinh giỏi quốc gia (Toán, Tin, Lý, Hóa, Sinh),
80
Olympic Quốc Tế (IMO, IOI, IPhO, IChO, IBO) và các kỳ thi Olympic trong khu
vực. Các khối chuyên của trường cũng luôn nằm trong top dẫn đầu 200 trường có
điểm trung bình thi đại học cao nhất Việt Nam từ lúc có bảng xếp hạng này.”[17]
Khối THPT chuyên Vật lý có được những ưu thế to lớn về chuyên môn mà ở
trường khác khó thể có. Đó là đội ngũ giáo viên có trình độ chuyên môn cao, có
những nhà khoa học đầu ngành, giàu kinh nghiệm và tâm huyết với sự nghiệp đào
tạo học sinh năng khiếu Vật lý, như: GS. Đàm Trung Đồn, PGS. Ngô Quốc Quýnh,
GS. TSKH. Nguyễn Châu, PGS. Lê Thanh Hoạch, PGS. Nguyễn Viết Kính, GS.
Bạch Thành Công, GS. Hà Huy Bằng ,GS. Nguyễn Quang Báu.... Không chỉ truyền
cho học sinh những kiến thức vật lý, các thầy cô còn thổi vào các em niềm đam mê
khoa học, đam mê Vật Lý giúp các em định hướng được con đường khoa học của
mình.
Trong các năm từ 1988 đến 2013 học sinh khối THPT chuyên Vật Lý và
trường THPT chuyên đã có 32 học sinh dành huy chương trong các kì thi Olympic
quốc tế: Cụ thể là 3 huy chương vàng, 13 huy chương bạc và 16 huy chương đồng.
Hàng năm sau khi qua các vòng loại của trường có khoảng 10 em được trực tiếp
tham gia đội tuyển HSG Vật lý Quốc gia và đã có rất nhiều em được giải cao. Có
những học sinh giỏi Vật Lý của trường đã từng được giải quốc gia, quốc tế nay trở
lại là giảng viên, giáo viên của trường. Đó chính là những bề dày truyền thống và
thế mạnh của đội ngũ giáo viên và học sinh của trường THPT chuyên KHTN -
ĐHQG Hà Nội.
1.3.2.Thực trạng công tác bồi dưỡng học sinh giỏi Vật Lý ở trường THPT chuyên
KHTN
1.3.2.1. Thuận lợi trong công tác bồi dưỡng học sinh giỏi ở trường THPT chuyên
KHTN
Học sinh thi vào những lớp chuyên Lý của trường vốn đã là các em yêu thích
say mê và phần nào có năng khiếu đối với môn Vật lý. Điểm chuẩn đầu vào được
đánh giá khách quan thông qua đợt thi riêng do trường tổ chức, trường luôn là một
trong những trường chuyên danh tiếng đứng đầu trong thành phố và cả miền bắc.
Do vậy, trường có cơ hội lựa chọn được các học sinh có nền tảng tốt, có động lực
phấn đấu và có lòng say mê với môn học. Đây chính là điều kiện cần cho một đội
80
tuyển trở thành hùng mạnh. Mỗi học kỳ trường luôn tổ chức các đợt thi đánh giá
công bằng cho học sinh các môn chuyên và thi đại học(Toán, Hóa, Anh văn và Lý
hoặc Sinh). Các học sinh của cả các lớp chuyên Toán, Hóa, Tin cũng rất yêu thích
và say mê môn Lý, thậm chí nhiều học sinh lớp chuyên Sinh học Vật Lý rất tốt.
Điều này khiến cho việc học môn Vật lý luôn là một trong những mục tiêu của học
sinh trường chuyên KHTN.
Công tác bồi dưỡng học sinh giỏi của trường luôn được sự quan tâm và hỗ
trợ của các trường Đại học Khoa học Tự nhiên, Đại học Quốc gia Hà Nội và Bộ
giáo dục và Đào tạo. Học sinh giỏi của trường không những được sự giảng dạy và
bồi đắp của các giảng viên kỳ cựu mà còn được sử dụng các trang thiết bị và thực
hành trong những phòng thí nghiệm của Trường đại học KHTN hoặc tham gia
nghiên cứu khoa học cùng các giảng viên của trường. Trường được dành số lượng
học sinh vào đội tuyển quốc gia tương đương số học sinh giỏi của tất cả các trường
THPT trên địa bàn thành phố.
Như vậy công tác dồi dưỡng học sinh giỏi Vật lý ở trường THPT chuyên có
rất nhiều thuận lợi so với nhiều trường THPT và THPT chuyên khác.
1.3.2.2.Một số yêu tố bất thuận lợi trong trong công tác bồi dưỡng học sinh giỏi ở
trường THPT chuyên KHTN
Mặc dù là trường THPT chuyên, nơi mà đa số các phụ huynh và học sinh
hướng tới với sự đam mê các môn chuyên theo sở thích của mình. Nhưng thực tế thì
mục tiêu trước mắt của nhiều phụ huynh, học sinh hiện nay lại là đến đây để có một
môi trường học tập tốt, điều kiện học tập và giáo viên giảng dạy tốt để các em có
thể đỗ đại học. Hơn nữa trong vài năm gần đây tỉ lệ học sinh có năng khiếu và đặc
biệt say mê Vật lý vào trường giảm hơn so. Nguyên nhân chính do hiện nay hệ
thống trường chuyên ở các tỉnh trên cả nước đã phát triển tương đối mạnh và trải
rộng, thu hút bớt nhiều phần nhân tài ở các vùng miền. Đồng thời cũng do đặc điểm
trung ở các thành phố, thị xã hiện nay học thi vẫn còn mang tính học gạo, các em
được ôn luyện, làm bài tập nhiều dẫn tới quen nên có kết quả thi tốt chứ không phải
là có năng khiếu, có phản xạ Vật lý và năng lực tư duy tốt.
Trong khi đó các đề thi học sinh giỏi các cấp hiện nay đòi hỏi các em tham
gia đội tuyển phải học cật lực, dành rất nhiều thời gian vào việc học đội tuyển nên
80
có thể bỏ bê các môn học khác. Điều này khiến không ít phụ huynh của những em
đã được chọn vào những vòng trong do dự là tiếp tục tham gia đội tuyển hay học
đều ba môn để thi đại học. Tất cả các lý do trên đã góp phần làm ảnh hưởng đến
chất lượng nguồn của đội tuyển học sinh giỏi Vật lý ở trường THPT chuyên KHTN.
Để phát huy những thuận lợi và khắc phục những yếu tố bất thuận lợi đó nhà
trường đã từng bước đổi mới công tác quản lý, thi cử để thu hút được nhiều hơn các
học sinh giỏi và thực sự say mê các môn chuyên tự nhiên, đồng thời không ngừng
đổi mới phương pháp dạy học, phương pháp tổ chức và bồi dưỡng học sinh giỏi để
80
nâng cao chất lượng các đội tuyển, trong đó có đội tuyển Vật lý .
KẾT LUẬN CHƯƠNG I
Trên đây, chúng tôi đã trình bày cơ sở lý luận của luận văn.Tìm hiểu về các
khái niệm và phương pháp bồi dưỡng học sinh giỏi, học sinh THPT chuyên, tìm
hiểu về bài tập Vật lý trong dạy học ở trường trung học phổ thông và cơ sở thực tiện
của công tác bồi dưỡng học sinh giỏi ở THPT chuyên. Cụ thể có thể tóm tắt bởi
những luận điểm chính sau:
Học sinh giỏi Vật lý là người có năng lực quan sát tốt, nắm vững bản chất của hiện
tượng Vật lý mong muốn khám phá các hiện tượng Vật lý và vận dụng tối ưu các
kiến thức thức Vật lý để giải quyết một hay nhiều vấn đề mới, bài tập mới. Học sinh
giỏi và học sinh giỏi Vật lý đều có đặc điểm là tích cực, chủ động, sáng tạo trong
học tập có năng lực tư duy phát triển. Giáo dục học sinh giỏi cũng có những mục
tiêu và phương pháp cụ thể và đặc trưng.
Bài tập Vật lý, là một phần hữu cơ của quá trình dạy học Vật lý vì nó cho phép hình
thành và làm phong phú các khái niệm Vật lý, phát triển tư duy Vật lý và thói quen
vận dụng kiến thức Vật lý vào thực tiễn. Trong quá trình dạy học ở trung học phổ
thông bài tập Vật lý đóng vai trò đặc biệt quan trọng. Ở mỗi góc độ, mỗi trường
hợp cụ thể bài tập Vật lý đều thể hiện có những vai trò đặc thù của mình. Bài tập
Vật lý cũng có nhiều mục đích sử dụng khác nhau, có các cách hướng dẫn khác
nhau nhưng cũng có thể xây dựng thành các bước giải . Để phát huy tốt vai trò quan
trọng của Bài tập Vật lý , khi giảng dạy hướng dẫn cho học sinh, giáo viên cần căn
cứ vào mục đích và đối tượng cụ thể để lựa chọn và kết hợp các phương pháp
hướng dẫn cho hợp lý.
Trường THPT chuyên KHTN trực thuộc trường Đại học Khoa học Tự nhiên- Đại
học quốc gia Hà nội là một trường chuyên có bề dày kinh nghiệm và đội ngũ giáo
viên có chuyên môn sâu. Trường có rất nhiều thuận lợi trong công tác bồi dưỡng
học sinh giỏi, cụ thể là được sử dụng nguồn lực giáo viên cùng cơ sở vật chất từ
các trường cấp trên và có một lượng học sinh ổn định theo cấu được trực tiếp tham
gia trong các kỳ thi học sinh giỏi quốc gia. Nhưng trái lại, vẫn còn những yếu tố
không thuận lợi đối với đội tuyển học sinh giỏi Vật lý. Do nhiều em học sinh giỏi
Vật lý có mục tiêu chính là thi đại học khiến niềm say mê Vật lý thực sự ở các em
80
giảm hơn, chất lượng đội tuyển bị ảnh hưởng.
Trước thực trạng đó đòi hỏi phải có những biện pháp đổi mới về phương
pháp dạy học và bồi dưỡng học sinh giỏi nhằm nâng cao chất lượng đội tuyển giúp
80
duy trì truyền thống giành nhiều giải cao của học sinh chuyên lý của trường.
CHƯƠNG 2
XÂY DỰNG HỆ THỐNG BÀI TẬP VÀ HƯỚNG DẪN HOẠT ĐỘNG GIẢI
BÀI TẬP CHƯƠNG CẢM ỨNG ĐIỆN TỪ – VẬT LÝ 11
2.1. Nội dung kiến thức chương Cảm ứng điện từ
2.1.1. Cấu trúc nội dung chương Cảm ứng điện từ
Nội dung chính của chương “Cảm ứng điện từ” trong chương trình Vật lý lớp
11 dạy ở trường THPT chuyên Tự nhiên có thể chia thành hai phần chính sau: Hiện
tượng cảm ứng điện từ và hiện tượng tự cảm.
Trong phần hiện tượng cảm ứng điện từ nghiên cứu bốn vấn đề: Từ thông, dòng
điện cảm ứng, suất điện động cảm ứng và bản chất của hiện tượng cảm ứng điện từ.
Giữa từ thông và dòng điện cảm ứng có mối quan hệ chặt chẽ với nhau, khi từ thông
qua khung dây biến thiên thì trong khung xuất hiện dòng điện cảm ứng. Giữa dòng
điện cảm ứng và suất điện động cảm ứng có mối quan hệ với nhau. Dòng điện Fuco
là trường hợp riêng của dòng điện cảm ứng. Suất điện động cảm ứng xuất hiện trên
đoạn dây dẫn chuyển động trong từ trường là trường hợp riêng của suất điện động
cảm ứng.
Trong phần hiện tượng tự cảm nghiên cứu các vấn đề chính: Từ thông riêng của
mạch kín, suất điện động tự cảm, năng lượng từ trường của ống dây tự cảm và mật
độ năng lượng từ trong ống dây và ứng dụng của hiện tượng tự cảm.
80
Sau đây là sơ đồ cấu trúc nội dung chương :
Sơ đồ 1.1.Cấu trúc nội dung chương Cảm ứng điện từ
CẢM ỨNG ĐIỆN TỪ
HIỆN TƯỢNG CẢM ỨNG ĐIỆN TỪ
HIỆN TƯỢNG TỰ CẢM
Ứng dụng Từ thông Dòng điện cảm ứng Suất điện động tự cảm
Suất điện động (SĐĐ) cảm ứng Từ thông riêng của mạch kín Năng lượng từ trường và mật độ năng lượng từ
Dòng điện Fucô
SĐĐ cảm ứng trong đoạn dây dẫn dẫn chuyển động
2.1.2. Phân tích sơ lược nội dung chương Cảm ứng điện từ
2.1.2.1. Hiện tượng cảm ứng điện từ
Từ thông:
“Đường cảm ứng từ là một đường cong vạch ra trong từ trường sao cho tiếp
tuyến tại mọi điểm của nó trùng với phương của vectơ cảm ứng từ tại những điểm
ấy, chiều của đường cảm ứng từ là chiều của vectơ cảm ứng từ”
Xét một diện tích dS đủ nhỏ trong từ trường sao cho vectơ cảm ứng từ qua diện tích
ấy có thể coi như bằng nhau tại mọi điểm. Ta đưa ra khái niệm từ thông gứi qua diện tích
dS là đại lượng có giá trị , trong đó là vectơ cảm ứng từ tại một điểm bất kì
80
trên diện tích ấy, là một vectơ nằm theo phương của pháp tuyến với diện tích đang
xét, có chiều là chiều dương của pháp tuyến đó, và có độ lớn bằng chính độ lớn của diện
tích đó.
Gọi là góc hợp bởi và (tức là góc hợp bởi và ), là hình chiếu
của lên phương pháp tuyến đó, là hình chiếu của lên mặt phẳng vuông
góc với đường sức từ, ta có:
nhọn hoặc tù có thể dương hoặc âm phụ thuộc vào góc
Số đường cảm ứng từ vẽ qua diện tích vuông góc với từ trường tỉ lệ với tích
. Như vậy, số đường cảm ứng từ qua cũng tỉ lệ với , tức là tỉ lệ với từ
thông.
Nếu muốn tính từ thông qua một diện tích S có kích thước lớn nằm trong một từ
trường bất kì, chia S thành các diện tích khá nhỏ dS sao cho trên mỗi phần tử ấy
vectơ cảm ứng từ là không đổi. Như vậy, từ thông gửi qua diện tích lớn là
Nếu diện tích S là phẳng nằm trong từ trường đều và vuông góc với các đường cảm ứng từ
thì:
Khi nói đến từ thông tức là muốn nói đến số đường cảm ứng từ đi qua một diện
tích nào đó, nhưng số đường cảm ứng từ thì luôn luôn dương, còn từ thông là
một đại lượng đại số, có thể âm hoặc dương phụ thuộc vào góc . Dựa vào công
thức trên, nếu thay đổi , , thì từ thông qua dS cũng sẽ thay đổi. Trong hệ
SI, đơn vị của cảm ứng từ B là Tesla, đơn vị của S là , lúc đó đơn vị của từ thông
là Wb.
+ Định lý Oxtrogradxki – Gauss (O-G) đối với từ trường
“Từ thông gửi qua bất kỳ mặt kín nào cũng bằng không”.
Biểu thức: (2)
Hay ở dạng vi phân: (3)
80
Các công thức (2) và (3) chứng tỏ đường sức của từ trường phải là đường khép kín.
Hiện tượng cảm ứng điện từ:
+ Thí nghiệm về hiện tượng cảm ứng điện từ (Thí nghiệm của Faraday)
Khi có chuyển động tương đối giữa nam châm và ống dây thì kim điện kế sẽ chỉ
lệch khỏi vị trí số 0, trong ống dây xuất hiện dòng điện . Dòng điện xuất hiện trong
ống dây gọi là dòng điện cảm ứng, và công làm chuyển động điện tích để tạo ra
dòng điện chạy trên ống dây tính cho một đơn vị diện tích được gọi là suất điện
động cảm ứng.
Suất điện động cảm ứng này có vai trò quan trọng trong đời sống hàng ngày, của
chúng ta.
Qua thí nghiệm đó, Faraday đã rút ra những kết luận tổng quát sau:
- Sự biến đổi từ thông qua mạch kín là nguyên nhân sinh ra dòng điện cảm
ứng trong mạch đó.
- Dòng điện cảm ứng ấy chỉ tồn tại trong thời gian từ thông gửi qua mach thay
đổi.
- Cường độ dòng điện cảm ứng tỉ lệ thuận với tốc độ biến đổi từ thông.
- Chiều của dòng điện cảm ứng phụ thuộc vào từ thông gửi qua mạch tăng hay
giảm.
Hiện tượng cảm ứng điện từ là hiện tượng khi từ thông qua một khung dây
dẫn kín biến thiên thì trong khung dây xuất hiện một dòng điện. Và dòng điện được
sản sinh ra gọi là dòng điện cảm ứng.
Dòng điện cảm ứng là dòng điện xuất hiện khi có sự biến đổi từ thông qua
mạch kín.
Các định luật về hiện tượng cảm ứng điện từ:
+ Định luật cảm ứng điện từ Faraday
Ban đầu, khi phân tích các kết quả thí nghiệm, Faraday đã phát biểu như sau :
Một lực điện động sinh ra bởi cảm ứngkhi từ trường quanh vật dẫn điện thay
đổi. Độ lớn của lực điện động cảm ứng tỉ lệ thuận với độ thay đổi của từ thông qua
vòng mạch điện.
Từ khái niệm từ thông, có thể phát biểu định luật Faraday một cách định lượng:
“Suất điện động cảm ứng luôn bằng về trị số, nhưng trái dấu với tốc độ biến thiên
80
của từ thông gởi qua vòng dây đó”
Trong hệ SI, suất điện động có đơn vị là V (vôn), còn tốc độ biến thiên của từ
thông theo thời gian được đo bằng Wb/s.
+ Định luật Len-xơ về chiều dòng điện cảm ứng
Dòng điện cảm ứng xuất hiện trong một mạch điện kín có chiều sao cho nó
chống lại sự biến đổi đã sinh ra nó.
Suất điện động cảm ứng trong đoạn dây dẫn chuyển động:
Xét thanh dẫn MN có chiều dài l, chuyển động với vận tốc v trong từ trường đều
có cảm ứng từ ( ), khi đó êlectron trong ống MN đang chuyển động trong từ
trường sẽ chịu tác dụng của lực Lorenxơ .
Giả sử cảm ứng từ có chiều đi vào như hình vẽ, lực Lorenxơ sẽ làm electron
chuyển động về phía M. Do đó, đầu N thừa electron, đầu M thiếu electron. Trong
đoạn MN xuất hiện điện trường hướng từ N đến M, gọi là điện trường cảm
ứng. Lúc này, electron chịu tác dụng của hai lực, lực điện trường và lực Lo-ren-
xơ . Sau một thời gian rất ngắn, hai lực này cân bằng nhau
Lực lạ ở đây chính là lực Lo-ren-xơ. Suất điện động cảm ứng trong mạch xuất
hiện là do lực Lo-ren-xơ gây ra. Bỏ qua điện trở của thanh nên ec=UMN=E.l. Vậy
theo công thức tính suất điện động cảm ứng của nguồn điện suy ra:
Thật vậy :
Dấu trừ đưa vào để nói về chiều của suất điện động cảm ứng.
Trường hợp thì ec=-Blv.sin
Quy tắc bàn tay phải:
“Đặt bàn tay phải hứng các đường cảm ứng từ, ngón cái choải ra 90o hướng
theo chiều chuyển động của đoạn dây, khi đó đoạn dây dẫn đóng vai trò như một
nguồn điện, chiều từ cổ tay đến bốn ngón tay chỉ chiều từ cực âm sang cực dương
của nguồn điện đó”.
80
Dòng điện Phucô (Foucault):
Dòng điện Phucô là dòng điện được sinh ra trong một khối vật dẫn điện khi cho
nó vào trong một từ trường biến đổi theo thời gian hay cho nó chuyển động cắt
ngang từ trường. Dòng điện Phucô là một dòng điện xoáy.
Như vậy dòng điện Phucô cũng là dòng điện cảm ứng, theo định luật Len-xơ, nó
cũng tạo ra một từ trường nhằm chống lại sự biến thiên từ thông đã gây ra nó.
Vì khối vật dẫn có điện trở R nhỏ nên cường độ dòng Phucô trong vật dẫn
Ic=ec/R
thường khá lớn. Mặt khác, suất điện động cảm ứng tỉ lệ thuận với tốc độ biến thiên
từ thông, nên nếu vật dẫn được đặt trong từ trường biến đổi càng nhanh thì cường
độ của dòng Phucô càng mạnh.
2.1.2.2. Hiện tượng tự cảm
Hiện tượng tự cảm và suất điện động tự cảm:
“Sự xuất hiện suất điện động cảm ứng trong mạch do sự biến thiên của từ thông
gây bởi dòng điện ở chính trong mạch đó được gọi là hiện tượng tự cảm. Và dòng
điện được sinh ra trong hiện tượng tự cảm gọi là dòng điện tự cảm”. Hiện tượng tự
cảm xuất hiện trong các mạch điện có dòng một chiều chạy qua hoặc khi ta đóng,
ngắt mạch điện, trong mạch điện xoay chiều hiện tượng tự cảm luôn luôn xảy ra.
Từ thông riêng của mạch do chính dòng điện trong cuộn dây đó tạo ra. Từ thông
tỉ lệ với cảm ứng từ do dòng điện trong mạch sinh ra, mà cảm ứng từ đó lại tỉ lệ
thuận với cường độ dòng điện của mạch. Do đó, từ thông tỉ lệ với dòng điện I, do đó
có thể đặt:
Trong đó L là hệ số tỉ lệ, phụ thuộc vào hình dạng, kích thước của mạch
điện và phụ thuộc vào môi trường vật chất mà ta đặt mạch điện vào, được
gọi là hệ số tự cảm hay độ tự cảm.
Theo định luật Faraday, biểu thức của suất điện động tự cảm là:
Năng lượng từ trường trong ống dây:
Cho một mạch điện gồm một nguồn điện không đổi có suất điện động e mắc nối
tiếp cuộn dây có độ tự cảm L với điện trở R và nối với nguồn qua khóa K. Giả sử
80
ban đầu mạch được đóng kín, trong mạch có dòng điện không đổi I. Khi ấy toàn bộ
điện năng do nguồn điện sinh ra đều biến thành nhiệt. Điều này là đúng khi trong
mạch có dòng điện không đổi, nhưng không đúng trong lúc đóng hoặc ngắt mạch.
Khi đóng mạch dòng điện trong mạch tăng từ 0 đến. Trong quá trình ấy, trong mạch
điện xuất hiện dòng điện tự cảm ngược chiều với dòng điện chính do nguồn
phát ra, làm cho dòng điện toàn phần ở trong mạch nhỏ hơn . Kết quả là
chỉ có một phần điện năng trong mạch biến thành nhiệt mà thôi. Trái lại, khi ngắt
mạch, dòng điện chính giảm đột ngột về 0. Do đó, trong mạch xuất hiên dòng điện
tự cảm, làm cho dòng điện toàn phần trong mạch lớn lên và giảm chậm lại, nhiệt
lượng tỏa ra trong mạch lúc này lớn hơn năng lượng do nguồn sinh ra.
Gọi R là điện trở trong toàn mạch, áp dụng định luật Ôhm cho mạch điện trong
quá trình dòng điện đang được thành lập, ta có:
Năng lượng do nguồn điện sinh ra trong khoảng thời gian dt, năng lượng này một phần tỏa thành nhiệt trong mạch còn một phần được tiềm tàng dưới dạng
năng lượng từ trường
Vậy, trong cả quá trình thành lập dòng điện, phần năng lượng của nguồn điện
được tiềm tàng dưới dạng năng lượng từ trường là:
Thực hiện phép tính tích phân, ta có:
Đây là công thức tính năng lượng dự trữ trong cuộn dây khi có dòng điện I chạy
qua.
Mật độ năng lượng từ trường
Năng lượng từ trường được phân bố trong không gian có từ trường. Có thể coi
cuộn dây điện thẳng dài vô hạn mà ta xét ở trên có từ trường được phân bố đều chỉ
trong thể tích của cuộn dây. Nếu gọi V là thể tích của cuộn dây thì ta có mật độ
năng lượng từ trường của cuộn dây điện là:
80
với và
Suy ra:
2.2. Mục tiêu chương Cảm ứng điện từ:
2.2.1. Kiến thức:
- Mô tả được thí nghiệm về hiện tượng cảm ứng điện từ.
- Viết được công thức tính từ thông qua một diện tích, nêu được đơn vị đo từ thông.
Nêu được cách làm biến đổi từ thông.
- Phát biểu được định luật faraday về cảm ứng điện từ và định luật Len-zơ về chiều
dòng điện cảm ứng.
- Viết được hệ thức và .
- Nêu được khái niệm dòng điện Fu-co. Tác dụng có lợi và khắc phục những ảnh
hưởng có hại của dòng Fu-co.
- Nêu được hiện tượng tự cảm.
- Nêu được khái niệm độ tự cảm và đơn vị đo độ tự cảm.
- Nêu được từ trường trong lòng ống dây có dòng điện chạy qua và mọi từ trường
đều mang năng lượng.
- Viết được công thức tính năng lượng của từ trường và mật độ năng lượng từ trong
lòng ống dây có dòng điện chạy qua.
2.2.2. Kỹ năng.
- Phân tích được các hiện tượng xảy ra trong một bài toán Vật lý cụ thể, phân tích
dữ kiện đề bài cho, xây dựng được mối liên hệ giữa các đại lượng vật lý.
- Giải thành thạo các bài tập định lượng và định tính về cảm ứng điện từ
- Biết bố trí và làm thí nghiệm về cảm ứng điện từ, tự cảm.
2.3. Phương pháp xây dựng và hướng dẫn giải hệ thống bài tập chương Cảm
ứng điện từ.
2.3.1. Phương pháp xây dựng hệ thống bài tập chương Cảm ứng điện từ.
Trước thực tế nội dung kiến thức trong các đề thi học sinh giỏi quốc gia của
nước ta hiện nay chủ yếu các bài tập tự luận thuộc mức độ tổng hợp, sáng tạo đòi
hỏi học sinh phải có khả năng tư duy logic, kỹ năng tổng hợp kiến thức, phát hiện
và giải quyết vấn đề trong những bài toán phức tạp. Trong đó có thể bao gồm các
80
bài tập khó thuộc chương Cảm ứng điện từ.
Đồng thời dựa vào đặc thù kiến thức của chương, căn cứ vào mục tiêu dạy
học, bồi dưỡng học sinh giỏi và đặc điểm của học sinh Chuyên Lý THPT chuyên
KHTN Hà Nội tôi xây dựng hệ thống bài tập chương Cảm ứng điện từ và phân loại
theo phương thức cho điều kiện và phương thức giải: Gồm 12 bài tập định tính và
15 bài tập định lượng, trong đó một số bài có hướng dẫn và một số bài để học sinh
tự giải.
2.3.2. Phương pháp hướng dẫn giải bài tập chương Cảm ứng điện từ
2.3.2.1. Phương pháp hướng dẫn giải bài tập định tính chương Cảm ứng điện từ
Các bài tập định tính trong chương cảm ứng điện từ cũng thuộc hai loại bài
tập định tính điển hình đó là giải thích hiện tượng và dự đoán hiện tượng. Các hiện
tượng Vật lý trong đó đều chịu sự chi phối của định luật cảm ứng điện từ Faraday
và định luật Len-xơ về chiều dòng điện cảm ứng. Ở đây trong qua trình hướng dẫn
giải bài tập tôi đã định hướng khái quát cho học sinh dẫn các em theo các bước giải
bài tập định tính, hình thành ở các em khả năng suy luận(luận ba đoạn) và kỹ năng
giải các bài tập định tính trong Vật lý. Cụ thể là sau khi phân tích đề bài tôi hướng
dẫn cho học sinh căn cứ vào những điều kiện cụ thể của đề bài, xác định những định
luật chi phối hiện tượng, dự đoán được hiện tượng gì xảy ra và xảy ra thế nào hoặc
từ đó giải thích được các hiện tượng.
2.3.2.2. Phương pháp hướng dẫn giải bài tập định lượng chương Cảm ứng điện từ
Những bài tập định lượng trong chương cảm ứng điện từ được sử dụng để
hướng dẫn cho học sinh giỏi ở đây chủ yếu là các bài có mức độ tổng hợp. Trong
những bài tập đó việc tìm kiếm lời giải phải thực hiện một chuỗi các lập luận logic,
biến đổi toán học qua nhiều mối liên hệ giữa những đại lượng đã cho, đại lượng
phải tìm với những đại lượng trung gian không cho trong dữ kiện bài toán. Bản thân
việc xác lập mối liên hệ trung gian đó là một bài tập cơ bản. Và do đó, muốn giải
được bài tập tổng hợp này buộc người học phải giải được thành thạo các bài tập cơ
bản, ngoài ra còn phải biết cách phân tích bài tập phức hợp để quy nó về các bài tập
đơn giản đã biết.
Kiểu hướng dẫn giải bài tập chủ đạo mà tôi sử dụng ở đây là định hướng khái
80
quát hóa kết hợp với hướng dẫn tìm tòi:
+ Định hướng tư duy của học sinh theo đường lối khái quát của việc giải quyết vấn
đề ở đây là hướng cho học sinh biết phân tích một bài toán phức tạp, tổng quát
thành các bài toán đơn giản, cụ thể. Thực chất là rèn cho học sinh biết cách suy luận
khái quát hóa “Từ một vấn đề rất khó, nếu chúng ta biết cách tập trung, gỡ rối từng
mảng thì chúng ta có thể đưa vấn đề ra ánh sáng. Ít ra, chúng ta có thể đặt vấn đề
một cách dễ hiểu hơn. Nói cách khác, chúng ta đã đi từng bước để khái quát hóa
vấn đề.”[ 11]
Đồng thời định hướng cho học sinh biết cách suy luận tổng quát hóa vấn đề “Ta gặp
một vấn đề F(w1, w2,…, wn) tại điểm các thông số đã là hằng nhất định. Giải xong
vấn đề này, ta tiến đến tổng quát hoá chúng cho các thông số wi bất định nằm trong
giới hạn nào đó”[11]. Thực chất là rèn luyện cho học sinh cách suy luận chuyển từ
việc khảo sát một tập hợp đối tượng sang một tập hợp đối tượng lớn hơn chứa tập
hợp đối tượng ban đầu. Sau khi làm một bài tập với một đại lượng trung gian cụ thể
xác định biết cách suy luận để chuyển sang một bài tập khác tổng quát hơn có đại
lượng trung gian bất kỳ.
+ Định hướng tìm tòi được sử dụng thông qua việc gợi mở để học sinh để các em tự
tìm cách giải quyết, tự xác định các hành động cần thực hiện để đạt các em có thể
chuyển từ các bài cụ thể, đơn giản sang các bài tập tổng quát phức tạp hơn.
Thật vậy ở đây tôi đã yêu cầu, định hướng để học sinh làm các bài toán cơ,
điện và điện từ đơn giản sau đó hướng các em tổng hợp kiến thức để làm những bài
toán tổng hợp khó hơn. Đồng thời gởi mở, định hướng để các em suy luận tổng quát
và tiến đến làm được cái bài toán có nhiều biến tổng quát hơn nữa.
Những bài tập sử dụng để hướng dẫn và xây dựng cho việc bồi dưỡng học
sinh giỏi ở chương này là những bài tập khó: Khó ở chỗ mỗi bài thường liên quan
đến nhiều mảng của Vật lý khiến khó ghép nối liên hệ giữa các mảng, liên quan
đến nhiều đại lượng Vật lý dẫn đến phải giải nhiều phương trình và sẽ khó trong
khâu xử lý toán học. Do vậy trong quá trình hướng dẫn giải các bài tập cho Học
sinh giỏi Vật lý trong quá trình hướng cho học sinh làm theo các bước của phương
pháp giải bài tập Vật lý tôi đã chú ý phân tích và hướng dẫn học sinh một số cách
80
làm cụ thể và chi tiết như sau:
+ Bước 1. Tìm hiểu đề bài: Đọc đúng đề bài, mô tả hiện tượng vật lí nêu trong đề
bài (có thể vẽ hình), xác định xem trong lớp hiện tượng vật lí đã cho có những đại
lượng vật lí nào đã cho, đại lượng nào cần tìm.
+ Bước 2. Xây dựng lập luận: Căn cứ vào các dữ kiện đề bài, các điều kiện ràng
buộc nhằm làm rõ bản chất Vật lý và diễn biến của các hiện tượng được mô tả trong
bài từ đó xác định các công thức các định luật biểu diễn mối quan hệ giữa các đại
lượng đã cho và cần tìm, từ đó lập các phương trình.
Bước 3. Luận giải: Trong quá trình giải các phương trình vốn liên quan đến các đại
lượng Vậy lý khác nhau thuộc nhiều mảng kiến thức thì mấu chốt để gỡ rối chính là
các định luật tổng quát nghiệm đúng trong nhiều quá trình Vật lý và các đại lượng
Vật lý chung tham gia, làm ảnh hưởng đến các giai đoạn của hiện tượng Vật lý. Ví
dụ định luật bảo toàn và chuyển hóa năng lượng áp dụng trong tất cả các quá trình
cơ, nhiệt, điện, quang, định luật bảo toàn điện tích…Một số đại lượng Vật lý
thường xuất hiện trong nhiề quuá trình cơ, điện, nhiệt, từ… như năng lượng, lực, áp
suất …Khi đồng nhất hoặc rút và thế các đại lượng đó trong các phương trình là
cách làm phổ biến và thuận tiện trong hầu hết các bài tập Vật lý từ dễ đến khó.
Bước 4. Biện luận: Phân tích kết quả cuối cùng để loại bỏ những kết quả không phù
hợp với điều kiện đầu bài tập hoặc không phù hợp với thực tế và kết luận
Thật vậy ở đây tôi đã yêu cầu, định hướng để học sinh làm các bài toán cơ,
điện và điện từ đơn giản sau đó hướng các em tổng hợp kiến thức để làm những bài
toán tổng hợp khó hơn. Đồng thời gởi mở, định hướng để các em suy luận tổng quát
và tiến đến làm được cái bài toán có nhiều biến tổng quát hơn nữa.
2.3.2.3. Mục đích của việc hướng dẫn giải bài tập chương Cảm ứng điện từ
Trong quá trình hướng dẫn giải bài tập của chương Cảm ứng điện từ đã định
hướng để học sinh hoàn thiện các kỹ năng giải bài tập Vật lý, biết kết hợp xây dựng
lập luận phân tích và lập luận tổng hợp. Mục tiêu cụ thể của quá trình hướng dẫn
các bài tập Vật lý ở đây là:
+ Rèn luyện kỹ năng giải bài tập Vật lý nói chung và kỹ năng giải bài tập chương
Cảm ứng điện từ nói riêng.
80
+ Củng cố và đào sâu kiến thức chương Cảm ứng điện từ.
+ Phát triển khả năng suy luận và và tư duy logic của học sinh, đó là điều kiện để
học sinh có thể giải được các bài toán khó, bài toán tổng hợp góp phần bồi dưỡng
học sinh giỏi môn Vật lý.
2.4. Xây dựng hệ thống bài tập và hướng dẫn hoạt động giải bài tập chương
2.4.1. Bài tập định tính
2.4.1.1. Bài tập có hướng dẫn
Bài 1:
Cho một vòng dây kim loại được treo bởi
sợi dây mềm cách điện như hình vẽ. Hiện
tượng gì sẽ xảy ra với vòng dây, nếu ta di
chuyển cực bắc của nam châm thẳng ra xa
vòng dây?
Hướng dẫn hoạt động giải bài tập 1:
Hoạt động của giáo viên Hoạt động của học Bài giải
sinh
1.Hướng dẫn học sinh tìm Đọc đề bài. 1.Tìm hiểu đề bài: Có nam
hiểu đầu bài: châm thẳng, vòng dây dẫn
kín và chuyển động của nam
? Căn cứ vào những dấu châm ra xa vòng dây.
hiệu đó các em hãy cho biết - Trả lời câu hỏi.
thấy chúng liên quan đến
hiện tượng Vật lý nào và - Ghi nhận. - Những dấu hiệu đó liên
chịu sự chi phối của những quan đến hiện tượng cảm
định luật nào? ứng điện từ, do vậy hiện
-GV sửa chữa, chốt lại. tượng chịu sự chi phối của
định luật cảm ứng điện từ
Fa-ra-dây và định luật Len-
2. Hướng dẫn học sinh xây -Lắng nghe, suy xơ.
dựng lập luận theo các nghĩ. 2. Lập luận:
đoạn. - Khi đưa thanh nam châm ra
80
- Trả lời câu hỏi xa vòng dây kín, từ thông
? Khi đưa thanh nam châm qua vòng dây giảm, theo
ra xa vòng dây kín, từ định luật cảm ứng điện từ
thông qua vòng dây thay -Lắng nghe, ghi trong vòng dây xuất hiện
đổi như thế nào…? nhận một dòng điện cảm ứng.
-Nhận xét chốt lại - Theo định luật Len-xơ,
? Hãy dùng định luật Len- - Trả lời câu hỏi dòng điện cảm ứng trong
xơ xác định chiều dòng vòng dây có chiều sao cho từ
điện cảm ứng và chiều cực -Lắng nghe, ghi trường mà nó sinh ra cùng
từ của từ trường cảm ứng nhận chiều với từ trường của nam
trong vòng dây? châm thẳng để chống lại sự
giảm của từ thông sinh ra nó.
- Vòng dây trở thành một
nam châm điện có từ trường
cùng chiều với từ trường
của nam châm thẳng ,
nghĩa là cực Nam của nam
châm điện đối diện với cực
Bắc của nam châm thẳng.
- Hai cực khác tên đối diện
nhau nên chúng hút nhau. -Yêu cầu học sinh hoàn 3. Kết luận: Trong vòng dây thành lập luận Hoàn thành lập luận xuất hiện dòng điện cảm - Lưu ý học sinh: Khi cần ứng, vòng dây bị hút, chuyển dự đoán hoặc giải thích động cùng chiều với nam một hiện tượng Vật lý các châm. em cần tìm ra quy luật nào,
định luật nào chi phối hiện - Lắng nghe, ghi
tượng và phương pháp lập nhận.
luận có thể tách thành các
đoạn và phải đảm bảo tính
80
logic.
Bài 2:
A
D
Cho mạch điện như hình vẽ, khi điều
A
chỉnh để điện trở R giảm thì hiện tượng gì
B
C
sẽ xảy ra trong khung dây dẫn kín ABCD
để cạnh ống dây? (Trong khung có xuất
hiện dòng điện cảm ứng không? Nếu có thì
R
dòng điện cảm ứng có chiều như thế nào?).
Biết mặt phẳng khung vuông góc với trục
ống dây.
A
D
A
B
C
R
ic Hướng dẫn hoạt động giải bài tập 2:
Hướng dẫn khái quát: Các em hãy căn cứ vào những điều kiện cụ thể của đề bài
Xác định những định luật chi phối hiện tượng, sau đó lập luận để đưa ra kết luận dự
đoán được hiện tượng gì xảy ra trong mạch điện, trong khung dây và xảy ra thế nào.
Bài giải:
1. Tìm hiểu đề bài: Ở đây có nguồn điện, biến trở,ống dây, khung dây kín ABCD và
có sự điều chỉnh biến trở để R của mạch điện giảm.
2. Xây dựng lập luận:
Đoạn 1: Khi điều chỉnh biến trở để R của mạch giảm đi, theo định luật Ôm
cường độ dòng điện trong mạch và trong ống dây tămg. Do vậy từ thông do từ
trường của dòng điện trong ống dây gửi qua khung dây ABCD tăng lên. Trong
khung dây ABCD xuất hiện dòng điện cảm ứng.
Đoạn 2: Theo định luật Lenxơ, dòng điện cảm ứng trong khung dây dẫn
ABCD có chiều sao cho từ trường mà nó sinh ra ( ) ngược chiều với từ trường
80
của dòng điện trong ống dây để chống lại sự tăng từ thông sinh ra nó( ).
Đoạn 3: Dựa vào cực của nguồn điện, ta xác định được chiều của dòng điện
trong ống dây là thuận chiều kim đồng hồ khi nhìn từ trên xuống. Áp dụng quy tắc
Nam thuận Bắc ngược, suy ra cực từ ở phía trên của ống dây là cực Nam. Áp dụng
quy tắc ra Bắc vào Nam ta xác định được chiều của từ trường của dòng điện
trong ống dây hướng từ trên xuống.
3. Luận giải: Từ trường ban đầu có hướng xuống thì từ trường cảm ứng có
hướng từ dưới lên. Áp dụng quy tắc nắm tay phải ta suy ra dòng điện cảm ứng trong
khung dây dẫn ABCD có chiều ABCD như hình vẽ.
4. Kết luận: Trong khi điều chỉnh R của mạch giảm thì trong khung dây ABCD xuất
hiện dòng điện cảm ứng có chiều ABCD.
2.4.1.2. Bài tập tự giải
Bài tập tự luận:
M
P
R tăng
Bài 1:
A
Hãy giải thích tại sao khi điều chỉnh biến
Q
trở cho điện trở của mạch R tăng thì hai
N
đoạn dây dẫn MN và PQ lại hút nhau?
A
B
A
Bài 2:
C
D
D
Hãy cho biết hiện tượng gì sẽ xảy ra
trong khung dây ABCD khi điều chỉnh
con trỏ của biến trở R sang phải?
Bài 3:
v2
Cho một vòng dây kim loại được treo bởi sợi dây mềm
cách điện như hình vẽ. Hiện tượng gì sẽ xảy ra với
vòng dây, nếu ta di chuyển của nam châm thẳng ra xa
v1
S N
vòng vòng dây với vận tốc v1 trong hai trường hợp:
a.Vòng dây cũng di chuyển cùng chiều với nam châm
với vận tốc v2< v1
b.Vòng dây cũng di chuyển cùng chiều với nam châm
80
với vận tốc v2= v1
Bài 4:
R
Q
Hãy xác định chiều dòng điện tự cảm do
E
ống dây gây ra trong mạch điện ở hình
M L
N
K
P
vẽ bên khi ta bắt đầu đóng khóa K?
Bài 5:
1
R
Cho mạch điện như hình vẽ bên hãy xác
B
A
định chiều dòng điện tự cảm do ống dây
2
K
gây ra qua đèn 2 trong thời gian ngắt
E
khóa K ?
Bài tập Trắc nghiệm:
Bài 6:
Ở hình vẽ bên khi K ngắt dòng điện tự cảm do
R
Q
ống dây gây ra, và dòng điện qua R
E
lần lượt có chiều như đáp án nào dưới đây:
A. Itc từ M đến N; IR từ Q đến M
M L
N
K
P
B. Itc từ M đến N; IR từ M đến Q
C. Itc từ N đến M; IR từ Q đến M
D. Itc từ N đến M; IR từ M đến Q
Đáp án A
Bài 7:
Ở hình vẽ bên khi dịch con chạy của điện trở C về
C
P
N
M
E
phía N thì dòng điện tự cảm
L
Q
do ống dây gây ra và dòng điện qua biến trở C lần
lượt có chiều:
A. IR từ M đến N; Itc từ Q đến P
B. IR từ M đến N; Itc từ P đến Q
C. IR từ N đến M; Itc = 0
D. IR từ N đến M; Itc từ P đến Q
C
80
Đáp án D
Bài 8:
1
R
C
B
A
Trong hình vẽ bên đáp án nào sau đây là đúng khi
2
xác định chiều dòng điện tự cảm do ống dây gây ra
K
E
và dòng điện qua đèn 2 trong thời gian đóng khóa
K:
A. Itc từ A đến B; I2 từ B đến C
B. Itc từ A đến B; I2 từ C đến B
C. Itc từ B đến A; I2 từ B đến C
D. Itc từ B đến A; I2 từ C đến B
Đáp án C
Bài 9: Hình vẽ nào xác định đúng chiều dòng điện cảm ứng trong đoạn dây dẫn
B
B
B
v
B
v
D.
v
chuyển động trong từ trường, biết dây dẫn vuông góc với mặt phẳng hình vẽ:
B.
v
Icư
Icư
Icư
Icư
C. A.
Đáp án A
Bài 10: Hình vẽ nào sau đây xác định đúng chiều dòng điện cảm ứng khi cho vòng
B
B
v
v
v
v
D.
C.
B.
Ic
Ic
Ic
B
Icư = 0 B
dây tịnh tiến với vận tốc trong từ trường đều:
ư
ư
ư
A.
Đáp án D.
2.4.2. Bài tập định lượng
2.4.2.1. Bài tập có hướng dẫn
Bài 1:
Trên mặt phẳng ngang có hai thanh kim loại
y
B
N
Ax, By, cách nhau một đoạn l = 50 (cm). Mắc
R
giữa A và B là nguồn một chiều E0 = 1,5V,
x
A
r0=0,5. Đồng thời vắt ngang trên đó một
M
thanh kim loại MN có điện trở RMN = r = 0,5
y
B
N
80
R
x
A
M
, khối lượng m = 50gam ban đầu được giữ
đứng yên và điện trở R = 0,75 sao cho
không có khả năng cản trở chuyển động của
thanh MN.. Tất cả được đặt trong từ trường
đều không đổi, có phương thẳng đứng, chiều
hướng lên trên, B = 0,5 (T)
a. Thả thanh MN không vận tốc ban đầu v0 =
0, thanh MN sẽ chuyển động như thế nào?
Nếu bỏ qua ma sát giữa MN với hai thanh.
b. Tìm suất điện động cảm ứng, cường độ
dòng điện qua MN và hiệu điện thế UMN khi
thanh có vận tốc v = 2 (m/s).
Hướng dẫn giải bài 1:
Định hướng:
Hoạt động của giáo viên Hoạt động của học sinh
+ Trước khi đưa ra Bài tập 1 giáo viên yêu + Làm bài tập 1.1 và 1.2
cầu học sinh làm hai bài tập cụ thể hơn 1.1
và 1.2 (Một bài tập điện từ và một bài tập
điện):
- Gọi hai học sinh lên bảng trình bày bài đã - Hai học sinh lên bảng trình bày bài
làm đã làm
- Giáo viên sửa chữa, nhận xét. - Lắng nghe, ghi nhận.
+ Giáo viên đưa ra đề bài 1 và yêu cầu học
sinh làm ( Gợi mở vấn đề bài này thực ra có + Làm bài tập 1
thể phân tích thành hai bài đơn giản khi giải
bài tập 1 ta có thể tổng hợp từ hai bài đơn
giản trên)
-Gọi một học sinh lên bảng trình bày bài làm
của mình. -Một học sinh lên bảng trình bày bài
-Giáo viên nhận xét sửa chữa. Giới thiệu lại làm.
cách phân tích đề bài.
80
+ Giáo viên chốt lại phương pháp : -Lắng nghe, ghi nhận
- Trong quá trình giải hệ phương trình hãy
chỉ ra ở bài này đại lượng nào là cầu nối giữa
hai giai đoạn(hai mảng kiến thức) cảm ứng + Trả lời câu hỏi.
điện từ và điện?
- Giáo viên nhận xét: Ở bài này suất điện
động cảm ứng ec được tạo ra do hiện tượng
cảm ứng điện từ ở giai đoạn trước(bài 1.1) là
một phần của mạch điện ở giai đoạn sau(bài + Lắng nghe, trao đổi với giáo viên
1.2) và ghi nhận.
- Lưu ý học sinh: Khi gặp những bài toán
phức tạp, tổng hợp kiến thức của nhiều phần
các em phải tìm hiểu đề bài để hiểu diễn biến
hiện tượng Vật lý và phân tích nó thành các
giai đoạn được lý giải bởi các kiến thức đơn
giản hơn đã biết. Từ đó lập các phương trình.
Khi giải hệ phương trình các em hãy chú ý
tìm nút tháo gỡ là các đại lượng Vật lý
chung giữa các giai đoạn diễn biến của bài
toán và chung giữa các mảng kiến thức.
Bài 1.1:
Trên mặt phẳng ngang có hai thanh kim
loại Ax, By, cách nhau một đoạn l = 50
(cm). Mắc giữa A và B là nguồn điện một
chiều E0. Một thanh kim loại MN ban đầu
được giữ vắt ngang như hình vẽ . Tất cả
được đặt trong từ trường đều không đổi, có
phương thẳng đứng, chiều hướng lên trên,
B = 0,5 (T):
a. Khi thả thanh MN không vận tốc ban
đầu v0 = 0, thanh MN sẽ chuyển động như
80
thế nào? Nếu bỏ qua ma sát giữa MN với
hai thanh.
b. Tìm suất điện động cảm ứng trong thanh
MN khi thanh có vận tốc 2m/s?
Bài giải 1.1:
Ic
B
y
N
B
y
N
I
I
A
x
x
A
M Hình a
M Hình b
a. Ban đầu khi MN đang được giữ đứng yên, do có dòng điện I chạy qua và đặt
trong từ trường nên thanh MN chịu lực từ tác dụng làm vật chuyển động (nhanh
dần) theo hướng như ở hình a. Xảy ra hiện tượng cảm ứng điện từ, theo Lenxo-
Faraday suy ra trong thanh xuất hiện dòng điện cảm ứng có chiều Ic ngược lại so
với I, gây ra lực từ tác dụng chống lại sự tăng tốc của thanh MN như ở hình b.
Như vậy ban đầu thanh chuyển động nhanh dần sau khi đạt vận tốc đủ lớn( = -
) thanh sẽ chuyển động thẳng đều.
b. Lúc thanh chuyển động với vận tốc v= 2 m/s thì suất điện động cảm ứng trong
thanh có độ lớn là:
Bài 1.2:
B
N
Cho mạch điện như hình vẽ bên: Nguồn
thứ nhất có suất điện động và điện trở
R
trong lần lượt là: Eo=1,5V, ro=0,5,
Eo,r0
Ec, r
nguồn thứ hai có có suất điện động và
M
A
điện trở trong lần lượt là: Ec=0,5,
r=0,5. Điện trở R =0,75. Hãy xác
định cường độ dòng điện đoạn qua MN
và hiệu điện thế UMN.
B
N
R
80
Eo,r0
Ec, r
Bài giải 1.2:
M
I2
A
I1 I
Giả sử chiều dòng điện như hình vẽ, áp dụng phương trình nút với điểm M ta có
(1) .
Trong đó áp dụng định luật Ôm đối với các đoạn mạch MRN, NBEoAM và NEcM,
B
N
ký hiệu UMN là U suy ra:
A
y
U = 3 - 3U => U = 0,75(V)
M
.
x
Dấu (-) chỉ dòng I2 có chiều từ M đến N ngược với chiều trên hình vẽ
Bài 2:
B
Trên mặt phẳng nghiêng có góc
N
nghiêng = 300 có hai thanh ray cách nhau
A
y
một đoạn = 50 (cm) cùng nghiêng 300
so với mặt phẳng ngang (hình vẽ bên) tại
M
A và B (với AB Ax) ta mắc một nguồn 1
x
chiều E0 = 2V, r0 = 0,5 . MN là một thanh
trượt có khối lượng m = 50 gam, có điện
trở rMN = r = 0,5 luôn vuông góc với hai
thanh ray. Bỏ qua điện trở của các thanh
ray, lấy g = 10 (m/s2).
Cảm ứng từ đều có chiều hướng từ trên
xuống dưới.
1. Cố định thanh MN ở vị trí BN = AM =
2m. Cho B giảm dần đều, cứ 10(s) thì giảm
0,1 (T)
a. Tính suất điện động cảm ứng xuất hiện
trong mạch kín, và dòng điện qua MN.
80
b. Tính lực từ tác dụng lên MN và AM,
đúng lúc B = 0,5 (T).
2. B không đổi, bằng 0,5 (T). Thả thanh
trượt tự do không ma sát.
a. Lúc vận tốc của thanh là v = 1 (m/s).
Hãy xác định dòng điện qua thanh và thế
hiệu UMN?.
b. Tìm vận tốc cực đại của thanh?.
3. Nếu B=0,5 T và hệ số ma sát là
tìm vận tốc cực đại của thanh?.
Định hướng:
Hoạt động của giáo viên Hoạt động của học sinh
+ Sau khi học sinh đã làm bài tập 1. Giáo + Học sinh làm bài tập 2.1
viên yêu cầu học sinh làm một bài tập cơ
2.1
+ Giáo viên đưa ra đề bài tập 2. + Đọc và tiếp nhận đề bài
+ Yêu cầu hoạc sinh thảo luận theo nhóm + Làm việc theo nhóm và cử đại diện
(Mỗi nhóm 4 người) để trả lời hai câu hỏi trình bày các câu trả lời.
sau:
+ Các em hãy nhận xét ở bài tập 2 và bài + So sánh với bài 1 và trả lời câu hỏi
tập 1 khi xét đến các thông số ảnh hưởng
đến chuyển động của thanh MN thì ở bài
tập nào tổng quát hơn? Vì sao?
+ Theo các em bài tập 2 liên quan đến mấy + Làm việc nhóm và trả lời câu hỏi
mảng kiến thức Vật lý? Có thể tổng hợp nó
từ mấy bài toán đơn giản hơn?
+ Hãy xác định tên các đại lượng Vật lý
chung liên kết trực tiếp giữa các mảng kiến + Làm việc nhóm và trả lời câu hỏi
thức ở bài này?
80
+ Giáo viên phân tích, nhận xét:
- Vấn đề ở bài tập 2 tổng quát hơn ở
bài tập 1 thể hiện ở hai thông số sau:
Góc nghiêng của hai thanh Ax,By
và hệ số ma sát giữa thanh
+ Lắng nghe. MN và hai thanh Ax,By
- Bài tập 2 có thể phân tích thành ba
giai đoạn, tổng hợp từ ba bài toán
điện từ, điện và cơ.
- Trong bài tập 2: Giữa bài toán điện
từ và bài toán điện có suất điện động
cảm ứng là đại lượng chung và liên
hệ trực tiếp. Giữa phần điện từ và
phần cơ có lực từ liên hệ trực tiếp và
ảnh hưởng đến nhau.
+ Lắng nghe, trao đổi + Như vậy các em hãy tổng hợp kiến thức
và phương pháp giải bài tập 1 và bài 2.1 để
+ Làm bài tập 2. tiến hành giải bài tập 2.
Bài 2.1:
Một vật khối lượng 2 kg được kéo trượt trên
mặt phẳng nghiêng bởi một lực theo phương
ngang như hình vẽ. Biết góc nghiêng .
a. Bỏ qua ma sát giữa vật và mặt phẳng
nghiêng, khi lực kéo có độ lớn F= 15 N vật sẽ
chuyển động như thế nào?
b. Cho hệ số ma sát trượt giữa vật và mặt phẳng
nghiêng là . Khi vật chuyển động thẳng
đều lên trên hãy tính độ lớn của lực kéo F .
80
Giải bài 2.1:
a. Khi bỏ qua ma sát vật chịu tác dụng của ba lực : lực kéo , trọng lực và
phản lực . => + + . Chọn trục ox song song với mặt phẳng
nghiêng và hướng lên:
Xét thành phần hình chiếu lên ox: , hướng lên.
, hướng xuống. Ta thấy: > => Vật sẽ
trượt lên nhanh dần .
b.
Áp dụng định luật II Niuton:
Khi vật chuyển động thẳng đều: =0
F(cos - ) = P( + cos)
80
Giải bài tập 2:
1.Thanh MN cố định
a. Khi B giảm, dòng điện cảm ứng xuất hiện trong hình chữ nhật, để chống
lại sự giảm từ thông dòng cảm ứng có chiều thuận kim đồng hồ ứng với
suất điện động ec chống lại suất điện động Eo trong mạch kín => Từ
thông:
Dòng điện IMN chạy từ M đến N
b. Lực tác dụng lên MN:
F1 = 0,50 N ( Đặt ở trung điểm của MN có tác dụng kéo MN lên trên
và ép MN vào mặt phẳng nghiêng)
Lực tác dụng lên AM : Lực này đặt ở trung điểm của AM, có phương song
song với MN (kéo AM vào trong) có độ lớn
= 1,728 (N)
2. a. Ta thử xét quan hệ giữa lực từ ban đầu tác dụng lên MN và trọng lượng của
thanh để biết lúc bắt đầu thả thanh trượt lên hay xuống.
Thành phần hình chiếu của trọng lực lên ox có độ lớn là
Psin = mgsin = 0,25N
Thành phần hình chiếu của lực từ ban đầu lên ox có độ lớn
= 0,325 N
Do F0 cos > mgsin nên lúc thả thì thanh trượt lên nhanh dần.
Dùng qui tắc bàn tay phải, suất điện động cảm ứng xuất hiện trong MN, có
cực dương phía M (trong mạch kín, ngược chiều với Eo)
80
ec= Blvsin120o= 0,5.1.0,5.sin120o=0,2165(V) (2.a.1)
Dòng điện qua thanh chạy từ M đến N
(2.a.2)
UMN = ec + Ir = 0,2165 + 1,7835 . 0,5 = 1,108V
Từ (1) => Khi vận tốc tăng dần thì ec tăng dần thay vào (2) suy ra I sẽ giảm dần và
khiến lực từ tác dụng lên MN giảm dần.
b. Khi thanh chuyển động đều thì thành phần hình chiếu lên phương ox của
lực từ và trong lực tác dụng lên thanh, cân bằng nhau
Fcos = mgsin
F = mgtan => BI = mgtan (2.b.1)
Thay (2.a.2) vào (2.b.1) => I =
(2.b.2)
= 30o với
vmax = 3,905(m/s)
3.
Khi có ma sát thanh chịu tác dụng của 4 lực, trong đó lực từ chống lại lực ma
sát trượt và thành phần trên mặt nghiêng (ox) của trọng lực.
Khi thanh chuyển động đều thì :
(1) Fcos = Psin + Fms
Chiếu tổng hợp lực lên trục oy vuông góc với mặt phẳng nghiêng, để tìm
biểu thức của lực ma sát, ta có : N = Fsin + Pcos
80
(Fsin + Pcos) (2) Fms =
Thay (2) vào (1)
F(cos - ) = P( + cos)
Thay P = 0,5, ta có (N)
Suy ra , mặt khác
= 0,5. I = 1,67 A Thay Eo=2, r0 + r = 1, B = 0,5,
2.5.1.1. Bài tập tự giải
Bài 1:
R
Hai thanh ray dẫn điện đặt thẳng đứng, hai đầu trên nối với
điện trở R = 0,5Ω; phía dưới thanh kim loại MN có thể trượt
B
N
M
theo hai thanh ray. Biết MN có khối lượng m = 10g, dài l =
25cm có điện trở không đáng kể. Hệ thống được đặt trong từ
trường đều B = 1T có hướng như hình vẽ, lấy g = 10m/s2, sau
khi thả tay cho MN trượt trên hai thanh ray, một lúc sau nó
đạt trạng thái chuyển động thẳng đều với vận tốc v bằng bao
nhiêu?
80
Đáp án: v= 0,8m/s
A
B
Bài 2: Cho khung dây dẫn ABCE là hình
vuông cạnh a = 8cm, DCNM là hình vuông
cạnh b=4cm (hình vẽ). Từ trường , biến
đổi theo quy luật B1=k1t, B2=k2t và có chiều
như( hình vẽ.3 )
D
C
E
Với k1= 10T/s; k2= -15T/s. Tính cường độ
dòng điện qua các đoạn khung dây. Biết một
cm chiều dài khung dây có điện trở 0,5Ω.
N
M
Đáp số: I1 =0, 374A (chiều: CBAE)
I2 =0,207 A (Chiều: CNMD)
I3 = 0,58A (Chiều: CD)
M
A
B
Bài 3. (Học sinh giỏi Vật lý Vĩnh Phúc 2012-2013):
Một dây dẫn cứng có điện trở không đáng kể, được
C
D
N
uốn thành khung ABCD nằm trong mặt phẳng nằm
1
ngang,có AB và CD song song với nhau, cách nhau
một khoảng l=0,5m, được đặt trong một từ trường
đều có cảm ứng từ B=0,5T hướng vuông góc với mặt phẳng của khung như hình 1.
Một thanh dẫn MN có điện trở R=0,5 có thể trượt không ma sát dọc theo hai cạnh
AB và CD.
a. Hãy tính công suất cơ học cần thiết để kéo thanh MN trượt đều với vận tốc
v=2m/s dọc theo các thanh AB và CD. So sánh công suất này với công suất tỏa
nhiệt trên thanh MN và nhận xét.
b. Thanh đang trượt đều thì ngừng tác dụng lực. Sau đó thanh còn có thể trượt thêm
được đoạn đường bao nhiêu nếu khối lượng của thanh là m=5gam?
Đáp số: a. = công suất tỏa nhiệt
80
b. S= 8 cm
Bài 4: Một ống dây dài = 31,4cm có 100 vòng, diện tích mỗi vòng S = 20cm2, có
dòng điện I = 2A chạy qua.
a) Tính từ thông qua mỗi vòng dây.
b) Tính suất điện động tự cảm trong cuộn dây khi ngắt dòng điện trong thời gian t
= 0,1s. Suy ra độ tự cảm của ống dây.
Đáp số: a) = 1,6.10-5 Wb ; b) e = 0,16V ; L = 0,008H.
Bài 5:
M
Một thanh kim loại MN dài l = 1m trượt trên hai
v
B
C
L,R
thanh ray đặt nằm ngang với vận tốc không đổi v =
N
2m/s. Hệ thống đặt trong từ trường đều B = 1,5T có
hướng như hình vẽ. Hai thanh ray nối với một ống
dây có L = 5mH, R = 0,5Ω, và một tụ điện
C = 2µF. Tính năng lượng từ trường trong ống dây
Đáp số: 0,09J
Bài 6: Dòng điện qua một ống dây không có lõi sắt biến đổi đều theo thời
gian.Trong thời gian 0,01s cường độ dòng điện tăng từ i1=1A đến i2=2A,suất điện
động tự cảm trong ống dây etc=20V.Hỏi hệ số tự cảm của ống dây và độ biến thiên
năng lượng từ trường trong ống dây .
Đáp số: L=0,2H; W=0,3J
Bài 7:
Một khung dây cứng phẳng diện tích 25cm2 gồm 10
vòng dây, đặt trong từ trường đều, mặt phẳng khung
B(T)
2,4.10-3
vuông góc với các đường cảm ứng từ. Cảm ứng từ
t(s)
biến thiên theo thời gian như đồ thị hình vẽ. Tính
z
0,4
suất điện động cảm ứng xuất hiện trong khung kể từ t
= 0 đến t = 0,4s:
80
. Đáp số: 1,5.10-4V
Bài 8:
i(A)
Một ống dây được quấn với mật độ 2000 vòng/m.
Ống có thể tích 500cm2 và được mắc vào mạch điện,
5
t(s)
sau khi đóng công tắc, dòng điện biến thiên theo thời
0
0,05
gian như đồ thị bên hình vẽ ứng với thời gian đóng
công tắc là từ 0 đến 0,05s. Tính suất
điện động tự cảm trong ống trong khoảng thời gian
trên:
Đáp số: 8π.10-2V
Bài 9:Một cuộn dây có 400 vòng điện trở 4Ω, diện tích mỗi vòng là 30cm2 đặt cố
định trong từ trường đều, véc tơ cảm ứng từ vuông góc với mặt phẳng tiết diện cuộn
dây. Tốc độ biến thiên cảm ứng từ qua mạch là bao nhiêu để cường độ dòng điện
trong mạch là 0,3A:
Đáp số: 1T/s
Bài 10: Cho mạch điện như hình vẽ,nguồn có
=6V,r=0,1 ,tụ có điện dung C=5 ,điện trở của
mạch R=2,9 .Điện trở thanh MN không đáng kể,MN
dài 1m: cảm ứng từ B=0,5T
a. Hãy tính điện tích của tụ, cường độ dòng điện
chạy trong mạch,lực từ tác dụng lên MN khi
MN đứng yên
b. Hãy tính điện tích của tụ, cường độ dòng điện
chạy trong mạch,lực từ tác dụng lên MN khi
MN chuyển động đều sang phải với vận tốc
20m/s,bỏ qua lực ma sát giữa MN và khung
c. Để tụ điện tích được một lượng điện tích là
Q=5,8.10-5C,thì thanh MN phải di chuyển về
phía nào? Với vận tốc là bao nhiêu?
Đáp số:a. I=2A,Q=2,9.10-5C,F=1N;
b.I=5,33A, Q =7,75.10-5C ,F=2,67N ;;
80
c.v=12m/s, sang phải.
Bài 11:
A
Thanh kim loại AB được kéo trượt đều trên
C1
hai thanh ray với vận tốc v = 10m/s, hai
thanh ray cách nhau một đoạn l = 0,5m, hệ
C2
B
thống đặt trong từ trường đều cảm ứng từ B
như hình vẽ. Mắc hai tụđiện C1 và C2 với C2
= 1,5C1 nối tiếp vào đầu hai thanh ray. Biết
hiệu điện thế hai đầu tụ C2 là 0,5V. Tính B.
Đáp số: B= 0,25 T.
Bài 12:
B
A
Hai thanh kim loại thẳng song song Ax và By
cách nhau một khoảng l, phía trên là một tụ có
điện dung C. Thanh MN có khối lượng m luôn
trượt theo phương thẳng đứng và tiếp xúc với 2
m,l
N
M
thanh.
Tìm gia tốc a của thanh MN bằng 2
phương pháp khác nhau:
y
x
a. Phương pháp động lực học
b. Phương pháp năng lượng.
Đáp số:
O
Bài 13:
Dây kim loại đồng chất tiết diện đều có điện trở
r trên một đơn vị dài. Thanh trượt MN cùng
bản chất, tiết diện như dây xoy, chuyển động
thẳng đều với vận tốc cùng phương với phân
M
N
giác của góc . MN luôn tiếp xúc với 2 cạnh
của góc. Từ trường đều không đổi vuông góc
y
x
với mặt phẳng xoy. Chứng tỏ rằng trong tam
giác có 1 dòng cảm ứng không đổi. Tính I2 theo r,
80
, v.
:
Đáp số
80
Vậy I không đổi
KẾT LUẬN CHƯƠNG II
Căn cứ vào mục tiêu bồi dưỡng học sinh giỏi, căn cứ vào đặc điểm kiến thức
và mục tiêu dạy học của chương cảm ứng điện từ cùng với đặc điểm của học sinh
giỏi Vật lý. Trong chương này chúng tôi đã xây dựng hệ thống bài tập và phương
pháp hướng dẫn giải bài tập chương cảm ứng điện từ - Vật lý 11. Hệ thống bài tập
gồm 12 bài tập định tính và 15 bài tập định lượng, mỗi dạng bài đó tôi định hướng
và hướng dẫn hai bài giúp rèn luyện cho học sinh khả năng suy luận logic, hiểu rõ
bản các hiện tượng Vật lý, rèn luyện các năng lực cần có của học sinh giỏi như tư
duy phân tích, tổng hợp và khái quát vấn đề, tổ hợp các kiến thức các (các bài tập)
đã biết để vạch ra cách làm cho các bài tập khó, có mức độ tổng hợp cao hơn.
Trong quá trình hướng dẫn giải bài tập về cảm ứng điện từ cho học sinh
chúng tôi đã sử dụng cách định hướng khái quát trương trình hóa, kết hợp hướng
dẫn tìm tòi. Cụ thể là hướng dẫn học sinh phương pháp giải bài tập Vật lý định tính
theo các bước phân tích đề bài tìm ra các quy luật, định luật chi phối hiện tượng và
tiến hành lập luận. Hướng dẫn giải các bài tập định lượng có mức độ khó, tổng hợp
liên quan đến nhiều mảng kiến thức bằng cách cho các em làm các bài tập đơn giản
tương ứng với từng mảng của bài tổng hợp. Sau đó hướng cho các em tổng hợp lại
và tự làm bài tập tổng hợp. Ngoài ra trên cơ sở bài tập có thông số cụ thể phát triển
vấn đề, hướng dẫn các em làm các bài tập tổng quát hơn. Hình thành ở học sinh khả
năng phân tích các bài toán phức tạp thành các bài tập đơn giản đã biết và sau đó
80
tổng hợp lại một cách chặt chẽ, chính xác để giải quyết bài tập phức tạp ban đầu.
CHƯƠNG 3
THỰC NGHIỆM SƯ PHẠM
3.1. Mục đích, đối tượng và phương pháp của thực nghiệm sư phạm
3.1.1. Mục đích của thực nghiệm sư phạm.
Thực nghiệm sư phạm nhằm mục đích kiểm tra tính đúng đắn của giả thuyết
khoa học mà đề tài đã nêu, có nghĩa là: Việc xây dựng được hệ thống bài tập đa
dạng, có mức độ tổng hợp, khó kết hợp với việc hướng dẫn hoạt động giải bài tập
theo các phương pháp có sự định hướng tư duy cho học sinh sẽ góp phần bồi dưỡng
học sinh giỏi Vật lý ở Trung học phổ thông chuyên.
Trong quá trình thực nghiệm sư phạm chúng tôi đã thực hiện các nhiệm vụ
sau:
- Tổ chức dạy học, hướng dẫn hoạt động giải bài tập cho học sinh và giao bài
tập cho các em làm.
- So sánh đối chiếu kết quả học tập ở lớp thực nghiệm và lớp đối chứng để sơ
bộ đánh giá hiệu quả của tiến trình hướng dẫn giải bài tập và hệ thống bài tập
chương cảm ứng điện từ.
- Đánh giá tính khả thi của việc hướng dẫn và cho học sinh làm hệ thống bài
tập đã đề ra. Trên cơ sở đó có những sửa đổi, bổ sung để hoàn thiện hệ thống bài tập
và phương pháp hướng dẫn hệ thông bài tập trên.
3.1.2. Đối tượng và phương pháp của thực nghiệm sư phạm
3.1.2.1. Đối tượng:
Chúng tôi tổ chức tiến hành TNSP ở lớp dự bị đội tuyển ở trường THPT
chuyên KHTN – Đại học KHTN Hà nội, tách ngẫu nhiên theo vần A,B,C tên các
em thành hai lớp nhỏ, gọi là lớp thực nghiệm và lớp đối chứng, về cơ bản trình độ
và điểm thi trung bình trong quá trình học đội tuyển trước đó của của các em là
tương đương nhau.
3.1.2.2. Phương thức thực nghiệm sư phạm
Quá trình thực nghiệm sư phạm, chúng tôi tiến hành song song, dạy ở lớp
thực nghiệm và dự giờ ở lớp đối chứng trong trong cùng khoảng thời gian, cùng
80
mục tiêu kiến thức là bài tập Vật lý ứng với chương cảm ứng điện từ khi bồi dưỡng
học sinh giỏi. Lớp thực nghiệm (Lớp 1) và lớp đối chứng (Lớp 2), mỗi lớp có 20
học sinh và có chất lượng học tập gần như tương đương nhau.
Ở lớp đối chứng chúng tôi vẫn tổ chức dạy với phương pháp như thông
thường cho học sinh làm các bài tập khó sưu tầm trong các đề thi học sinh giỏi và
giáo trình giành cho học sinh chuyên lý về cảm ứng điện từ rồi sau đó chữa bài và
nhận xét.
Lớp thực nghiệm chúng tôi tổ chức giảng dạy theo phương án đã được xây
dựng. Trong qúa trình dạy chú ý quan sát thái độ, mức tham gia của từng học sinh.
Trong lúc các em làm bài tập và một học sinh lên bảng trình bày bài tôi đã xem vở
của một số em để theo dõi và đánh giá cách lập luận và giải bài tập của các em.
Sau khi kết thúc quá trình hướng dẫn và cho học sinh làm bài tập, chúng tôi
đã tiến hành cho hai lớp kiểm tra 90 phút, để kiểm tra đánh giá sự rèn luyện kĩ năng
và hiệu quả giải các bài tập khó của học sinh
Ngoài việc cho học sinh làm bài kiểm tra, chúng tôi còn tổ chức trò chuyện,
trao đổi với học sinh để biết ý kiến của các em học sinh về quá trình hướng dẫn và
làm bài tập.
Cuối đợt thực nghiệm sư phạm, chúng tôi tiến hành phân tích kết quả thu được
theo các phương pháp thống kê toán học
3.1.2.3 Tiêu chí đánh giá kết quả thực nghiệm sư phạm
Để có thể đạt kết quả cao khi nghiên cứu về hệ thống và hướng dẫn giải bài
tập Vật lý và TNSP được tốt thì cần phải có những tiêu chí để đánh giá kết quả
TNSP. Sau đây là một số tiêu chí đánh giá kết quả TNSP.
- Độ chính xác của kết quả kiểm tra khi không sử dụng hệ thống và hướng
dẫn giải bài tập vật lý.
- Độ tin cậy kết quả kiểm tra khi sử dụng hệ thống và hướng dẫn giải bài tập
- So sánh kết quả số lượng điểm kiểm tra cao trong học tập của học sinh khi
80
không và có sử dụng hệ thống bài tập và hướng dẫn giải bài tập Vật lí.
Bảng 3.1: Bảng xếp loại học tập theo các mức: Giỏi, Khá, Trung bình, Yếu, Kém
Xếp loại Điểm
Giỏi 9 - 10
Khá 7 - 8
Trung bình 5 - 6
Yếu 3 - 4
Kém 0
3.1.2.4. Thời gian tiến hành thực nghiệm
Đầu học kì 1 năm học 2013-2014. Lý do chọn thời gian thực nghiệm sư
phạm này là vì trên lớp chính các em học sinh lớp chuyên lý vừa mới học xong
phần lý thuyết . Các em đã đạt được những yêu cầu cơ bản của mục tiêu chương
theo yêu cầu của chương Cảm ứng điện từ - Vật lý 11 và mở rộng kiến thức theo kế
hoạch giảng dạy chung của trường THPT chuyên KHTN, chưa có sự ôn tập riêng
cho học sinh giỏi và các em học sinh chưa tự ôn tập được nhiều để từ đó kết quả
thực nghiệm có độ chính xác cao hơn.
3.2. Tiến hành thực nghiệm sư phạm
Khảo sát, học lực và đặc điểm của học sinh giỏi ở hai lớp thực nghiệm và đối
chứng là tương đương nhau trước khi tiến hành TNSP
Tiến hành dạy ở lớp thực nghiệm trong thời gian 12 tiết, 3 tuần.Trong đó 2
tiết đầu hướng dẫn bai bài tập định tính 1.1 và 1.2 và cho các em tự làm bài 3, giao
những bài tập còn lại yêu cầu các em về nhà làm, hai tiết sau hướng dẫn cho các em
làm hai bài tập định lượng như cách đã đề ra, sau đó giao nhiệm vụ về nhà các bài
còn lại. Bốn tiết tuần tiếp theo sẽ hướng dẫn giải đáp một số bài mà các em yêu cầu.
Tuần cuối 2 đầu giải đáp thắc mắc và thảo luận về phương pháp giải của hệ thống
bài tập, 2 tiết sau làm bài kiểm tra.
80
Tiếm hành chấm bài kiểm tra, phân tích và xử lý kết quả.
3.3. Kết quả và xử lý kết quả
3.3.1. Phân tích định tính diễn biến các giờ học trong quá trình TNSP
Trong quá trình thực hiện thực nghiệm việc hướng dẫn và sử dụng hệ thống
bài tập chương cảm ứng điện từ, qua quan sát và trao đổi chúng tôi nhận thấy những
điểm chính như sau:
Khi hướng dẫn học sinh làm hai bài định tính sau khi giao bài tập 1 đa số các
em đã nhanh chóng nhận ra nam châm và cuộn dây sẽ hút nhau, nhưng các em
thấy rất khó trình bày lập luận. Nhưng sau khi được hướng dẫn bài 1và được định
hướng làm bài 2 thì rất nhiều em đã phát hiện và trình bày đúng lập luận ở bài 2
và đến những giờ trao đổi giải đáp thì chỉ có vài ba em chọn sai đáp án bài 9. Khi
làm bài kiểm tra ở lớp thực nghiệm đa số các em đã lập luận chặt chẽ câu 1.
Khi hướng dẫn phần bài tập định lượng các em rất hào hứng khi phát hiện bài
tập 2 là tổng hợp của bài tập 1 và bài 2.1 và do vậy rất tích cực và chủ động khi làm
bài tập này. Dẫn đến kết quả là ở bài kiểm tra các em đã làm các bài toán có mức độ
khó, tổng hợp đã tốt lên nhiều.
Nhìn chung, các mục tiêu đặt ra trong quá trình bồi dưỡng với các kết quả sau khi
bồi dưỡng đều đã thực hiện được, cụ thể:
- Không khí học tập của học sinh sôi nổi, các em tích cực chủ động tham gia xây
dựng bài cũng như mạnh dạn nêu ý kiến của mình để cùng nhau thảo luận trao đổi
trong mỗi giờ học, đặc biệt là giờ giải đáp thảo luận.
- Khả năng vận dụng lý thuyết vào việc giải bài tập được nâng cao rõ rệt. Nắm
vững được nhiều phương pháp giải và vận dụng một cách hiệu quả trong mỗi bài
toán.
- Tư duy vật lý, tư duy lí luận của học sinh được phát triển thể hiện ở việc các em
đã giải quyết được nhiều bài tập dành cho học sinh khá giỏi một cách nhanh chóng
và chính xác.
- Kỹ năng quan sát, phân tích, của học sinh đối với các hiện tượng vật lí được nâng
cao, từ đó có thể mở rộng bài toán và vận dụng kiến thức vào các vấn đề mới, thể
hiện nhiều em đã làm tốt cả câu 3 trong bài kiểm tra, câu này mới lạ hơn các bài tập
80
trong hệ thống bài tập cho về nhà .
3.3.2. Phân tích bài kiểm tra
Để có căn cứ đánh giá chúng tôi đã soạn thảo và tổ chức cho học sinh làm bài
kiểm tra viết, với thời gian 90 phút sau khi kết thúc quá trình hướng dẫn và cho học
sinh làm hệ thống bài tập " Chương cảm ứng điện từ". Nội dung bài kiểm tra bao
gồm 4 bài tập trong chương, 1 bài định tính và 3 bài đinh lượng, các bài định lượng
cũng là các bài tập tổng hợp, khó liên quan đến cơ , điện và cảm ứng điện từ và đòi
hỏi học sinh phải vận dụng những kỹ năng được rèn luyện trước đó để giải. Kết quả
bài kiểm tra là căn cứ đánh giá năng lực giải quyết vấn đề, tư duy logic, phân tích ,
tổng hợp khả năng giải được các bài tập khó đã được rèn luyện của học sinh (Đề và
đáp án bài kiểm tra được trình bày ở phụ lục 1).
Sau khi tổ chức cho học sinh làm bài kiểm tra, chúng tối tiến hành chấm bài
và xử lí kết quả thu được từ bài kiểm tra theo phương pháp thống kê toán học: tính
các tham số đặc trưng , S2, S, V, vẽ đồ thị phân bố tần suất và tần suất luỹ tích hội
tụ lùi.
+ Trung bình cộng :
+ Với xi là điểm số, fi là tần số, N là tổng số học sinh của lớp.
+ Phương sai S2 và độ lệch chuẩn S là các tham số đo mức độ phân tán của các
số liệu quanh giá trị trung bình cộng. S càng nhỏ chứng tỏ số liệu càng ít phân
tán.
,
+ Hệ số biến thiên V (chỉ mức độ phân tán của các giá trị xi xung quanh giá
): trị trung bình cộng
+ Tần suất wi và tần suất tích lũy hội tụ lùi
Tần suất:
80
+ Tần suất tích lũy hội tụ lùi: w = (≤ i )
Bảng 3.2. Bảng thống kê điểm số
Điểm số Số Lớp HS 0 1 2 4 5 6 7 8 9 10 3
Đối chứng 20 0 0 0 0 4 4 4 5 3 0 0
Thực 20 0 0 0 0 0 0 6 5 6 3 0 nghiệm
Bảng 3.3. Bảng các tham số thống kê
Tổng số HS Lớp S2 S V%
20 Đối chứng 6,95 1,946 1,395 20,1
20 Thực nghiệm 8,30 1,169 1,081 13,0
Bảng 3.4. Bảng thống kê số học sinh đạt từ điểm xi trở xuống
Số % học sinh đạt từ điểm xi trở xuống Tổng số
1 2 3 4 5 6 7 8 9 10 HS Lớp
Đối 20 0 0 0 0 20 40 60 85 100 - chứng
Thực 20 0 0 0 0 0 0 30 55 85 100 nghiệm
Từ bảng số liệu trên đây chúng tôi vẽ đồ thị đường phân bố tần suất và tần
80
suất lũy tích hội tụ lùi cho lớp thực nghiệm và lớp đối chứng
Hình 3.1. Đồ thị đường phân bố tần suất
Hình 3.2. Đồ thị đường phân bố tần suất lũy tích hội tụ lùi
+ Đánh giá kết quả:
- Điểm trung bình lớp thực nghiệm (8,3) cao hơn lớp đối chứng (6,95)
- Hệ số biến thiên giá trị điểm số của lớp thực nghiệm (13%) nhỏ hơn lớp đối
chứng (20,1%) nghĩa là độ phân tán về điểm số quanh điểm trung bình của lớp thực
80
nghiệm là nhỏ hơn lớp đối chứng.
- Đường tần suất lũy tích hội tụ lùi của lớp thực nghiệm nằm bên phải và ở
phía dưới của đường tần suất lũy tích hội tụ lùi của lớp đối chứng, chứng tỏ chất
lượng nắm vững và vận dụng kiến thức ở lớp thực nghiệm tốt hơn lớp đối chứng.
Song vấn đề đặt ra là các kết quả khác nhau có thực sự là do phương pháp dạy học
mới đem lại hay không? Các số liệu có đáng tin cậy không? Để trả lời câu hỏi này,
tôi áp dụng bài toán kiểm định trong thống kê toán học.
TN và S2
DC (S2
TN là
Trước hết, phải kiểm định sự khác nhau của các phương sai S2
DC là phương sai của lớp đối chứng)
phương sai của lớp thực nghiệm, S2
Chọn mức ý nghĩa = 0,05.
TN và S2
DC ở hai mẫu là không có
Giả thiết H0: “Sự khác nhau giữa các phương sai S2
TN = S2
DC”
ý nghĩa” nói cách khác “phương sai ở các tổng thể chung là bằng nhau: S2
TN
Giả thiết H1: Sự khác nhau của hai phương sai của hai lớp là có ý nghĩa (S2
DC)
# S2
Đại lượng kiểm định F:
Tra giá trị F từ bảng phân phối F, ứng với mức và các bậc tự do là: fTN =
f1= 20 ; fDC = f2= 20, ta có:
Vậy, vì F < F nên ta chấp nhận giả thiết H0 : Sự khác nhau giữa các phương
sai là không có ý nghĩa, tức là phương sai của tổng thể chung là bằng nhau, chứng
tỏ hai lớp thực nghiệm và đối chứng có chung một tiền đề xuất phát.
+ Tiếp theo, ta kiểm định sự khác nhau của hai giá trị trung bình
với phương sai bằng nhau.
Chọn mức ý nghĩa = 0,05.
Giả thiết H0: Sự khác nhau của hai giá trị trung bình là không có ý nghĩa hay
. Tức là chưa đủ để kết luận phương pháp mới tốt hơn phương pháp cũ.
Giả thiết H1: Sự khác nhau của hai giá trị trung bình là có ý nghĩa. Tức là
phương pháp mới có hiệu quả hơn phương pháp cũ.
Đại lượng kiểm định: t =
80
Do đó, t = 3,42
Vì NTN = NDC = 20 nên ta tra trong bảng phân bố chuẩn với mức ý nghĩa là
= 0,05 ta tìm được t = 2,086
Vậy t > t nên bác bỏ giả thiết H0 và chấp nhận giả thiết H1, tức là sự khác
nhau giữa hai giá trị trung bình là có ý nghĩa. Tức là phương pháp mới có hiệu quả
hơn phương pháp cũ.
3.4. Đánh giá chung về thực nghiệm sư phạm
Sau khi tiến hành thực nghiệm sư phạm và xử lý số liệu, chúng tôi nhận ra
một số nhận xét sau:
x- Học sinh ở lớp thực nghiệm có khả năng suy luận, trình bày lập luận tốt hơn,
nâng cao được kỹ năng giải bài tập Vật lý và vận dụng một cách khoa học trong
việc giải các bài toán khó, bài toán tổng hợp.
- Kết quả kiểm tra cho thấy ở lớp thực nghiệm điểm trung bình cao hơn ở nhóm đối
chứng.
- Tỉ lệ học sinh đạt điếm khá giỏi ở các lớp thực nghiệm cao hơn và tỉ lệ học sinh
và trung bình của các lớp thực nghiệm thấp hơn nhóm đối chứng.
- Đồ thị đường các lũy tích về tỉ lệ học sinh đạt dưới điếm xi của lớp thực nghiệm
nằm về bên phải và phía dưới đồ thị các đường lũy tích tương ứng của nhóm đỗi
chứng,điều đó chứng tỏ kết quả học tập của lớp thực nghiệm tốt hơn lớp đối chứng.
Về hệ số biến thiên V của các lớp thực nghiệm cũng nhỏ hơn các nhóm đối chứng,
điều đó chứng tỏ mức độ phân tán quanh giá trị trung bình cộng của các lớp thực
nghiệm nhỏ hơn, nghĩa là chất lượng của các lớp thực nghiệm đồng đều hơn, ổn
định hơn so với các đối chứng.
- Trên cơ sở đó, có thể kết luận rằng: Việc sử dụng hệ thống các bài tập và hướng
dẫn giải các bài tập vật lý trong quá trình bồi dưỡng HSG cho học sinh lớp thực
nghiệm đã mang lại hiệu quả cao, học sinh thu nhận kiến thức chắc chắn và sâu
hơn, kỹ năng phân tích và giải bài tập Vật lý khó tổng hợp nhiều vấn đề thành thạo
hơn. Học sinh phát hiện và dự đoán chính xác các hiện tượng trên cơ sở phân tích
các biểu hiện bên ngoài tìm ra quy luật chi phối và lập luận chặt chẽ để đưa ra được
kết luận đúng. Qua đó đã phát triển được tư duy vật lý, tư duy lý luận ở các em. Kết
quả thể hiện các em làm bài kiểm tra đã giải được các bài tập vật lý khó, đặc trưng
80
trong các đề thi học sinh giỏi một cách thành thạo.
KẾT LUẬN CHƯƠNG 3
Thông qua một số tiết học ít ỏi của quá trình thực nghiệm sư phạm với số lượng
học sinh hạn chế, chưa đủ để khẳng định giá trị phổ biến của hệ thống bài tập và
phương pháp hướng dẫn giải bài tập mà chúng tôi đưa ra. Tuy nhiên, với những kết
quả bước đầu thu được có thể chứng tỏ: Việc xây dựng được hệ thống bài tập
chương “Cảm ứng điện từ - Vật lý 11” gồm nhiều dạng bài, có mức độ khó, tổng
hợp nhiều mảng kiến thức. Kết hợp với việc hướng dẫn hoạt động giải bài tập theo
các phương pháp có sự định hướng tư duy cho học sinh sẽ góp phần bồi dưỡng học
sinh giỏi Vật lý ở Trung học phổ thông chuyên.
Thật vậy sau khi tổ chức dạy thực nghiệm ở lớp thực nghiệm và dự giờ ở lớp
đối chứng, tiến hành phân tích đánh giá định tính các giờ học và phân tích bài kiểm
tra, cơ bản chúng tôi đã thu được những kết luận sau.
- Học sinh tích cực tham gia các hoạt động dạy học, hào hứng trả lời câu hỏi
của giáo viên, thích thú với việc nhận ra phương pháp phân tích các bài toán phức
tạp tổng hợp thành các bài toán cơ bản đã biết .
- Sử dụng hệ thống bài tập và hướng dẫn giải đã giúp khả năng làm các bài tập
khó cả định tính lẫn định lượng của học sinh tăng một cách đáng kể. Điều đó khẳng
định việc xây dựng hệ thống bài tập Vật lý và phương pháp hướng dẫn giải bài tập ở
đây đă có hiệu quả cao trong việc phát triển tư duy lý luận, rèn luyện được kỹ năng
giải bài tập Vật lý, kích thích được lòng say mê Vật lý và chinh phục những bài tập
khó của học sinh giỏi.
- Nhìn chung hệ thống bài tập và phương pháp giải các bài tập chương “Cảm
ứng điện từ” đã xây dựng là khả thi, đã nâng cao hiệu quả của công tác bồi dưỡng
HSG môn Vật lý.
-Tuy nhiên do thời gian thực nghiệm có giới hạn nên đề tài chỉ minh
chứng trong phạm vi hẹp. Để đề tài thành công trong phạm vi rộng hơn cần
phải có những yêu cầu cao hơn. Cụ thể: cần phải tiến hành thực nghiệm trên
nhiều đôi tượng học sinh giỏi hơn, thực hiện nhiều bài khiểm tra đánh giá
hơn, từ đó điều chỉnh và bổ sung hệ thống bài tập sao cho phù hợp và đạt hiệu
80
quả cao hơn trong bồi dưỡng việc HSG.
KẾT LUẬN VÀ KHUYẾN NGHỊ
1. Kết luận
Với nhiệm vụ nghiên cứu của đề tài đặt ra chúng tôi đã nghiên cứu cơ sở lí luận
của việc bồi dưỡng học sinh giỏi Vật lý và học sinh THPT chuyên. Nghiên cứu cơ
sở lí luận về bài tập Vật lý, về việc sử dụng bài tập và hướng dẫn giải bài tập Vật lý
ở THPT. Tìm hiểu thực trạng của công tác bồi dưỡng học sinh giỏi Vật lý ở trường
THPT chuyên Tự nhiên.
Từ đó xây dựng được hệ thống bài tập chương “Cảm ứng điện từ- Vật lý 11”
gồm nhiều dạng bài, có mức độ khó, tổng hợp nhiều mảng kiến thức. Kết hợp với
việc hướng dẫn hoạt động giải bài tập theo các phương pháp có sự định hướng tư
duy như hướng dẫn tìm tòi và định hướng khái quát chương trình hóa góp phần bồi
dưỡng học sinh giỏi Vật lý ở Trung học phổ thông chuyên.
Phương pháp hướng dẫn giải bài tập đặc trưng mà tôi sử dụng ở đây là:
- Tách các bài toán khó, tổng hợp nhiều mảng kiến thức thành các bài toán đơn
giản cho học sinh làm trước, sau đó mới định hướng cho các em làm bài tổng hợp
cần giải sau, cách làm này rèn luyện được cho các em khả năng tổng hợp và phân
tích vấn đề.
- Định hướng cho học sinh phát hiện ra cách giải quyết những bài toán có các
tham số tổng quát hơn sau khi đã được làm những bài tương tự có các tham số cụ
thể.
- Hướng dẫn và phân tích phương pháp giải bài tập định tính chương cảm ứng
điện từ cho học sinh.
Cuối cùng thông qua quá trình thực nghiệm sư phạm, kết quả đạt được của học
sinh giỏi sau khi được hướng dẫn và giải hệ thống bài tập chương “Cảm ứng điện từ
- Vật lý 11” theo cách làm của đề tài cho thấy:
- Hệ thống bài tập và hướng dẫn giải bài tập đã phát huy được tính tích cực của học
sinh, kích thích niềm say mê môn học của học sinh chuyên Vật lý.
- Hệ thống bài tập và hướng dẫn giải bài tập đã phát triển ở các em các năng lực tư
duy: lý luận, phân tích, tổng hợp, khái quát hóa…giúp các em giải chính xác các
80
bài tập khó.
Với kết quả như trên, đề tài đã đạt được mục đích đề ra và khẳng định được giả
thuyết khoa học ban đầu.
Tuy nhiên qua quá trình nghiên cứu đề tài chúng tôi cũng nhận thấy, đề tài này
còn một số điểm cần khắc phục như sau:
- Do thời gian bố trí bồi dưỡng có hạn, trên cơ sở đa số các học sinh đã hình
thành và phát triển các năng lực suy luận, phân tích, tổng hợp …, giải thành thạo
các bài toán khó giáo viên cần tổ chức thêm giờ tự học để các học sinh trao đổi và
giải đáp cho nhau những bài tập được giao về nhà trong hệ thống bài tập mà một số
bạn chưa thực sự làm tốt và thông hiểu. Làm như vậy các bạn giỏi hơn sẽ có cơ hội
thể hiện năng lực trình bày, phân tích của mình tốt hơn, tự tin hơn đồng thời các bạn
chậm hơn sẽ hiểu rõ hơn những vấn đề cần đạt được.
- Do điều kiện thời gian và khuôn khổ của luận văn TNSP mới chỉ tiến hành
trên lượng khách thể nhỏ, nếu được tiến hành trên diện rộng hơn nữa, thực hiện ở
nhiều trường chuyên có những đặc điểm tương tự sẽ đánh giá khách quan và chính
xác hơn giả thuyết của đề tài.
2. Khuyến nghị
Phương pháp dùng hệ thống bài tập được đề cập trong luận văn có thể áp
dụng đối với hầu hết các kiến thức vật lý trong chương trình học ở các trường
THPT Chuyên hiện nay. Tuy nhiên cách triển khai và tổ chức cụ thể cần căn cứ vào
đặc điểm của từng trường và giai đoạn bồi dưỡng cho học sinh giỏi.
Chúng tôi hy vọng rằng: Đề tài sẽ góp phần vào việc đổi mới phương pháp
dạy học ở trường THPT chuyên và công tác bồi dưỡng học sinh giỏi Vật lý.
Chúng tôi cũng rất mong được sự các thầy cô giáo trong trường THPT, THPT
chuyên, các nhà sư phạm và các giáo viên Vật lý góp ý kiến cho đề tài của chúng tôi
hoàn thiện hơn nữa, tạo điều kiện cho chúng tôi mở rộng sang phần nội dung khác
trong chương trình vật lý phổ thông, góp phần nâng cao chất lượng dạy học Vật lý
nói chung và công tác bồi dưỡng học sinh giỏi Vật lý THPT nói riêng trong giai
80
đoạn hiện nay.
TÀI LIỆU THAM KHẢO
1. Nguyễn Quang Báu (2007), Bài tập Vật lí nâng 11, Nxb Đại học Sư phạm.
2. Ban tổ chức kỳ thi Olympic truyền thống 30/4(2010), Tuyển tập đề thi
Olympic 30-4, lần thứ XVI- Môn Vật lí, Nhà xuất bản Đại học Sư Phạm.
3. Bộ Giáo dục và Đào tạo (ngày 15/02/2012), Quy chế Tổ chức và hoạt động của
trường trung học phổ thông chuyên, Ban hành kèm theo Thông tư số: 06/2012/TT-
BGDĐT.
4. Lương Duyên Bình (2009), Vật lí đại cương, Nhà xuất bản Giáo dục Việt Nam.
5. Phạm Kim Chung (2011), Bài giảng chuyên đề, phương pháp dạy học Vật lý.
Đại học Giáo dục - Đại học Quốc gia Hà Nội.
6. Phạm Kim Chung (2006), Bài giảng phương pháp dạy học Vật lí ở trường
Trung học phổ thông, Đại học Giáo dục - Đại học Quốc gia Hà Nội.
7. Vũ Cao Đàm (2011) Phương pháp luận nghiên cứu khoa học, Nhà xuất bản
Giáo dục Việt Nam.
8. Nguyễn Phú Đồng (2013), Bồi dưỡng học sunh giỏi Vật lý, tập 1, Nhà xuất
bản Tổng hợp thành phố Hồ Chí Minh.
9. Phạm Minh Hạc (1996), Tuyển tập Tâm lý học J.Piaget, NXB Giáo dục.
10. Nguyễn Thị Mỹ Lộc ( 2009), Tâm lý học giáo dục, Nhà xuất bản Đại học quốc
gia Hà Nội.
11. Võ Quang Nhân, Trần Thế Vỹ (2004), “ Các phương pháp suy luận và sáng
tạo”,http://vietsciences.free.fr/thuctap khoahoc/renluyen_sangtao/khainiemhoa.htm.
12. Nguyễn Đức Thâm (2002), Phương pháp dạy học vật lí ở trường phổ thông.
Nxb Đại học Sư phạm.
13. Nguyễn Đức Thâm (2002), Tổ chức hoạt động nhận thức cho học sinh ở
trường phổ thông trong dạy học vật lí. Nxb Đại học Sư phạm.
14. Phạm Hữu Tòng (1989), Phương pháp dạy bài tập vật lí, Nxb Giáo dục.
15. Phạm Hữu Tòng (1994), Bài tập về phương pháp dạy bài tập vật lí, Nxb Giáo
dục.
16. Đỗ Ngọc Thống (2007), “Bồi dưỡng học sinh giỏi ở một số nước phát triển”,
http://edu.hochiminhcity.gov.vn.
17.Đặng Đình Tới (24/1/2011), “ Nơi hội tụ của những tài năng trẻ yêu thích Vật
80
lý”, hsgs.edu.vn.
PHỤ LỤC
ĐỀ KIỂM TRA CHẤT LƯỢNG – THỜI GIAN 90 PHÚT
Bài 1(2 điểm):
C
D
Cho khung dẫn ABCD kín ban đầu hình
vuông đặt cạch thanh dây dẫn MN trên
mặt phẳng ngang có dòng điện đi qua
A
B
trong từ trường đều có phương thẳng
N
M
đứng chiều hướng lên như hình vẽ. Nếu
kéo khung thành hình chữ nhật ngày
càng dẹt đi(Chu vi khung không thay
đổi). Hãy dự đoán hiện tượng gì sẽ xảy
ra với khung dây và thanh MN ?
ĐA: Đoạn AB và MN hút nhau.
Bài 2(3 điểm): Trên một mặt phẳng nghiêng
góc α so với mặt phẳng ngang có hai dây dẫn
thẳng song song điện trở không đáng kể nằm
dọc theo đường dốc chính của mặt phẳng
nghiêng ấy.Đầu trên của hai dây dẫn nối với
điện trở R. Một thanh kim loại MN =l, điện
trở r, khối lượng m, đặt vuông góc với hai dây
dẫn nói trên, trượt không ma sát trên hai dây
dẫn ấy. Mạch điện đặt trong từ trường đều,
cảm ứng từ B có phương thẳng đứng và
hướng lên.
1) Thanh trượt xuống dốc, xác định chiều
của dòng điện cảm ứng chạy qua
R?(1đ)
2) Chứng minh rằng ngay lúc đầu thanh
kim lọai chuyển động nhanh dần đến
80
một lúc chuyển động với vận tốc không
đổi. Tính giá trị vận tốc khôi đổi
ấy.(2đ)
Đáp số:
Bài 2(3 điểm):
L
Cho mạch điện như hình vẽ, cuộn dây có độ tự
K
cảm L, điện trở thuần không đáng kể. Các tụ có
C2
điện dung C1 và C 2. Người ta đóng khóa K.
a. Tìm dòng điện cực đại qua cuộn dây. (1,5đ) b.
C1
b.Tìm hiệu điện thế cực đại trên hai bản của tụ
U
C1.(1,5đ)
Đáp số: a.
.
b.
Bài 3(2 điểm):
Một thanh kim loại dài 1m trượt trên hai thanh ray
nằm ngang như hình vẽ.Thanh kim loại chuyển động
đều với vận tốc v=2m/s.Hai thanh ray đặt trong từ
trường đều như hình vẽ.Hai thanh ray được nối với
một ống dây và một tụ điện.Ống dây có hệ số tự cảm
L=5mH,có điện trởR=0,5 . Tụ điện có điện dung
C=2 .Cho B=1,5T. Cho biết điện trở của thanh MN
và hai thanh ray có giá trị không đáng kể
a. Chiều của dòng điện cảm ứng qua ống dây?
b. Năng lượng từ trường qua ống dây?
c. Năng lượng điện trường trong tụ điện?
d. Điện tích của mà tụ tích được là bao nhiêu?
ĐS: a.Q N; b. Wtừ=0,09J;
80
c. Wđiện=9.10-6J; d. Q=6.10-6F